Green Notes

July 21, 2017 | Author: Aze Rabanal | Category: Will And Testament, Marriage, Foreclosure, Offer And Acceptance, Judgment (Law)
Share Embed Donate


Short Description

Civil Law Green Notes...

Description

in CIVIL LAW Prepared by: Atty. Roney Jone P. Gandeza event of my operation.”

REVIEW NOTES IN

CIVIL LAW

1. Abe is to be operated on for a cancerous tumor. Before the operation, he delivers an envelope to his friend, Rey. The envelope contains a letter saying, “I realize my days are numbered, and I want to give you this check for P1 million in the event of my death from this operation.” Rey cashes the check. The surgeon performs the operation and removes the tumor. Abe recovers fully. Several months later, Abe dies from a heart attack that is totally unrelated to the operation. Upon Abe’s death, the administrator of his estate immediately commences an action to recover from Rey the P1 million. Will the suit prosper? Yes. The donation to Rey is a donation mortis causa, not inter vivos. It is a mortis causa donation because Abe intended it to take effect upon his death, i.e. “in the

from

this

That Abe died from a cause unrelated to the operation does not detract from the fact that Abe’s death is the operative cause that would have conveyed ownership of the Php1 million given to Rey. Since the donation was not expressed in accordance with the formalities of wills, the donation is void.

ATTY. RONEY JONE P. GANDEZA UNIT 10 2nd FLOOR BBCCC BLDG. NO. 56 COOPERATIVE ST. COR. ASSUMPTION RD. BAGUIO CITY __________ LAW PROFESSOR COLLEGE OF LAW UNIVERSITY OF THE CORDILLERAS HARRISON ROAD, BAGUIO CITY __________

death

2. Rey drafted and properly executed a notarial will. Assume the following clauses in his will and the following events: (a)Rey’s will provides, “I leave my 2011 white Mitsubishi Montero with Plate No. AFB 346 to my friend Abe.” At the time of Rey’s death, Abe has already died, leaving one child, Zandro. (b)Rey’s will provides, “I leave to my friend, Abe, my 2011 white Mitsubishi Montero with Plate No. AFB 346.” Just prior to Rey’s death, he sold the Montero. Discuss fully each situation and the legal effect of the events on the testamentary provisions. (a) The bequest to Abe is rendered void by Abe’s predecease. The property devolves to Rey’s heirs in intestacy. Zandro, Abe’s son, has no right to the property because a voluntary heir, such as Abe, who dies before the testator transmits nothing to his heirs. (par. 1, Art. 856, CC) (b) The sale by Rey of property bequeathed to constitutes a revocation of bequest in favor of Abe. The

I press toward BAR OPERATIONS 2013 the mark for the

prize of the high calling of God in Christ Jesus. - Philippians 3:14

BARRISTERS’ CLUB

Page 1 of 67

the Abe the law

in CIVIL LAW Prepared by: Atty. Roney Jone P. Gandeza grants Rey this right because of the ambulatory character of a will. Abe has no right to question the sale because the right of an instituted heir (legatee or devisee) is merely inchoate.

psychologically incapacitated to enter into marriage under Philippine law. Under French law, the marriage is voidable. Is the marriage also voidable in the Philippines?

3. Debtor owed Creditor P400, 000. The debt is the subject of a lawsuit, and the court awards Creditor a judgment of P400, 000 against Debtor. To satisfy the judgment, the sheriff levies on Debtor’s family home in Baguio City valued at P500, 000. Debtor opposes the levy on the allegation that the family home is exempt from execution. Judgment for whom?

The marriage is void. The rule is: “valid there, valid here”; “void there, void here”; “voidable there, voidable here.” However, the case falls under one of the exceptions, Article 36 of the Family Code which refers to a void marriage by reason of the psychological incapacity of a spouse. 5. What is an agency coupled with an interest? An agency coupled with an interest is a relationship created for the benefit of the agent. The agent actually acquires a beneficial interest in the subject matter of the agency. Under these circumstances, it is not equitable to permit a principal to terminate the agency at will. Hence, this type of agency is irrevocable.

Judgment for Creditor. Under Article 160 of the Family Code, if judgment is rendered against the owner or owners of a family home, and the creditor has reasonable ground to believe that the value of the family home is in excess of P300, 000 or P200, 000, as the case may be, the creditor may apply for an order directing the sale of the family home.

6. Aristotle needs P100, 000. Socrates agrees to lend him the money, but not without security. Consequently, Aristotle delivers some of his jewelry to Socrates and signs a power of attorney giving Socrates the power, in case he fails to repay the loan, to sell the jewelry as his agent for the best price that can be obtained and to pay out of the proceeds the unpaid amount of the loan, giving any surplus to him. Having obtained the money, Aristotle later tells Socrates that he revokes the power to sell.

In the case presented, the value of the Debtor’s family home is in excess of P300, 000, it being located in an urban area; hence, the same may be sold at public auction to satisfy the judgment against him. The foregoing rule applies even if the increase in value of Debtor’s family home resulted from improvements introduced by Debtor. 4. Baldomero, a Filipino, marries Antonette, his French girlfriend, in Paris. At the time of the marriage, Baldomero is

Is the agency revoked?

I press toward BAR OPERATIONS 2013 the mark for the

prize of the high calling of God in Christ Jesus. - Philippians 3:14

BARRISTERS’ CLUB

Page 2 of 67

in CIVIL LAW Prepared by: Atty. Roney Jone P. Gandeza No. Under the law of agency, the power is not revoked. Aristotle has no right to terminate the agency at will. This is because the agency is coupled with an interest. If Aristotle dies, the power is still not affected.

sued for breach of Judgment for whom?

contract.

Judgment for Seller. Buyer’s telegram of December 16, refusing to the terms stated in Seller’s letter of December 8, varied the number of computer units, and was therefore a counter-offer. A counter-offer is in law a rejection of the original offer. On December 8, the Seller declined to fulfill the Buyer’s order, thus the negotiations between the two parties was closed. The Buyer’s attempt to fall back on the Seller’s original offer by the telegram of December 19, therefore, created no rights against the Seller. 9. A and B, Filipinos, both women, enter into a same-sex marriage in Denmark, where such a marriage is valid. Is the marriage also valid here?

7. A and B orally contract for the sale of A’s house for P1 million. A writes B a letter confirming the sale by identifying the parties and the essential terms – description and location of the property, price, place of payment, and method of payment – and signs the letter. Is the sale enforceable by court action? No. the sale was made orally. However, A has made a written memorandum of the oral land contract. Because A signed the letter, he can be held to the oral contract by B. Because B has not signed a written contract or memorandum, B can plead the Statute of Frauds as a defense, and A cannot enforce the contract against him.

The general rule enunciated in the first paragraph of Article 26 of the Family Code should apply, under which marriages valid where celebrated are also valid here. The case does not fit into any of the exceptions enunciated in the foregoing provision of Article 26 of the Family Code. Therefore, same sex marriage is valid here if valid where celebrated.

8. Seller, in reply to an inquiry from Buyer, sent a letter dated December 8 stating terms upon which he would sell 100 to 300 computer units of a certain brand at a certain price. On December 16, Buyer sent a telegram to Seller ordering 90 computer units on those terms. On December 18, Seller sent a telegram to Buyer rejecting the order. The next day Buyer sent Seller a telegram stating, “Please enter an order for 150 computer units as per your letter of December 8.” Seller refused the order, and Buyer

However, it would seem that the case falls under one of the exceptions to the application of the proper foreign law i.e., it runs counter to an important public policy of the forum – marriage should be between a man and a woman. Because of this important public policy, same sex marriage is void here. 10. A politician promises a newspaper editor that he will

I press toward BAR OPERATIONS 2013 the mark for the

prize of the high calling of God in Christ Jesus. - Philippians 3:14

BARRISTERS’ CLUB

Page 3 of 67

in CIVIL LAW Prepared by: Atty. Roney Jone P. Gandeza pay the editor P50, 000 if the editor will publish a false story indicating that the politician’s opponent is gay. Suppose the editor publishes the story, is he entitled to collect the amount promised?

oral agreement that is supports. The writing need not be single document. (A) is incorrect because the Statute of Frauds requires only that the party to be charged sign the written agreement or note or memorandum. The signature requirement of the Statute of Frauds is met if the writing contains the signature of the party to be charged or that of his agent. Therefore, both parties do not need to sign the writing to be enforceable under the Statute.

No, because the agreement is contrary to public policy. Considering that the parties are in pari delicto, the law will afford them no relief against each other. The editor may even be held criminally and civilly liable for the libelous article. 11.

The Statute of Frauds:

(B) is incorrect because the P500.00 threshold under the Statute of Frauds is relevant only to contracts for the sale of goods. (Art. 1403, 2(d), CC) Thus, a contract for services is not within the Statute even if the consideration is P500.00 or more, as long as it can be performed in a year.

a. Mandates that both parties sign the written contract or note or memorandum evidencing their agreement. b. Applies to all contracts under which payment of P500.00 is to be made.

(D) is incorrect because the Statute of Frauds contains no such provisions. The Statute of Frauds requires certain agreements to be evidenced by a writing signed by the party sought to be bound. Despite its name, the Statute does not set forth the elements of fraud as applicable to contractual relationships.

c. Has no requirement that the agreement of the parties be reduced to one document. d. Sets forth the elements of fraud as applicable to inducing another party to enter into a contractual relationship.

12. Why does the law impose an obligation in the case of quasicontract?

(C) is correct. The Statute of Frauds has no requirement that the parties’ agreement be reduced to one document. The Statute requires the essential terms of certain contracts to be evidenced by a writing, a note or memorandum to be enforceable. (Art. 1403, CC) Any writing will suffice as long as it contains every essential term of the

To prevent unjust enrichment. In an action for unjust enrichment, “recovery is based upon the universally recognized moral principle that one who has received a benefit has the duty to make a restitution when to retain such benefit would be unjust.”

I press toward BAR OPERATIONS 2013 the mark for the

prize of the high calling of God in Christ Jesus. - Philippians 3:14

BARRISTERS’ CLUB

Page 4 of 67

in CIVIL LAW Prepared by: Atty. Roney Jone P. Gandeza 13. Pedro was seriously in a vehicular accident. A bystander called Dr. Garcia to render medical treatment while Pedro was unconscious. Dr. Garcia sent Pedro a bill for the reasonable value of his medical services. Pedro refuses to pay. Judgment for whom?

The requirement is not absolute. As a rule, the filing of an action for legal separation requires a “cooling off” period for six months. (Art. 58, FC) However, when the ground alleged os one of those falling under RA 9262, also known as the Anti-Violence Against Women and their Children Act of 2004, there is no such “cooling off” period because the courts are mandated to proceed with the hearing of the case as soon as possible. (Sec. 19, Ra 9262)

Judgment for Dr. Garcia on the basis of a quasi-contract. In the problem presented, there was a benefit bestowed to Pedro, a benefit which is compensable under the law; otherwise, Pedro will be unjustly enriched at the expense of Dr. Garcia. 14. Abe took his car to a carwash station and asked to have it washed. While it is being washed, Abe went to a nearby mall for two hours. In the meantime, one of the workers at the car wash had mistakenly hand-waxed the car. When Abe came back, he was presented with a bill for a wash job and a hand wax. Is Abe liable to pay for the hand wax job?

16. A and B are Filipino overseas workers in Syria. They fell in love and decided to get married in accordance with Syrian law. On the day of the wedding, A fell ill and could not make it to the ceremony. Upon advice of his Syrian lawyer, he requested his best man to stand as proxy during the wedding. The marriage was celebrated in accordance with Syrian law and valid there as such. Is the marriage valid here in the Philippines?

No. the doctrine of quasicontract does not apply when there is a contract between the parties. Although there was a benefit bestowed to Abe, Abe did not receive an unjust benefit because the hand wax job was not contemplated by his agreement with the owner of the car wash station.

Yes. Under Article 26 of the Family Code, a marriage valid where celebrated is valid in the Philippines. To this general rule, the law enumerates exceptions, but a marriage by proxy is not one of those enumerated. Hence, the marriage is valid. Expressio unius est exclusion alterius.

15. Article 58 of the Family Code expressly provides that an action for legal separation shall in no case be tried before six months shall have elapsed since the filing of the petition. Is this requirement absolute?

17. H, a Filipino, marries W, an American, in New York. At the time of the marriage, H was psychologically incapacitated to enter into marriage, although the incapacity manifested itself five years after the wedding.

I press toward BAR OPERATIONS 2013 the mark for the

prize of the high calling of God in Christ Jesus. - Philippians 3:14

BARRISTERS’ CLUB

Page 5 of 67

in CIVIL LAW Prepared by: Atty. Roney Jone P. Gandeza Assuming that the marriage is valid in New York, is it also valid here?

estate which shall be taken from the free portion. Applying the foregoing provisions, Anastacia is therefore entitled to a legitime of P50, 000, while Magdalena is entitled to a legitime of P25, 000 which is to be taken from the free portion. The remainder of the estate constitutes the free portion which goes to Anastacia being the instituted heir.

No. The rule is:”valid there, valid here,” except those prohibited under Arts. 35 (1), (4), (5) and (6), 36, 37 and 38 of the Family Code. The case falls squarely under one of the exceptions enunciated in Article 26 of the Family Code; namely, a void marriage under Article 36. 18. Two second cousins, Filipinos, got married in Japan where such a marriage is void. Is the marriage valid here?

20. X and Y orally agreed that X would lease office space to Y at fixed rent as soon as the current lessee of the premises vacates in two years time. Almost two years later, and before he took possession of the premises, Y learned of a much more advantageous opportunity and established his office elsewhere. To compel Y to perform, X brought suit to enforce the agreement.

No. The rule is:”valid there, valid here.” Hence, “void there, void here” also. This notwithstanding the fact that had the marriage been solemnized here, it would have been perfectly valid. 19. Abe is the legitimate son of the widow, Anastacia. While single, Abe made out a will naming his mother as sole heir to an estate valued at P100, 000. Later, Abe married Magdalena.

Y’s strongest argument in his suit would be: a. Mistake. b. Impossibility of performance c. Statute of Frauds. d. Autonomy of Contracts. The Statute of Frauds is Y’s strongest argument. To be enforceable, the Statute requires certain agreements to be evidenced by a writing signed by the party to be charged, including agreements that can be performed within one year from its making. Because the lease in question is to be performed after one year, the Statute of Frauds is Y’s strongest argument. 21. Tiger, Phil, and Rory are coowners of a parcel of land. Tiger sells his undivided share in the

If Abe died while married to Magdalena without changing his will, would his entire estate of P100, 000 go to his mother, Anastacia? Article 889 of the Civil Code expressly provides that legitimate parents or ascendants consist of one-half of the hereditary estates of their children and descendants. Pursuant to Articlr 893, if the testator leaves no legitimate descendants, but leaves legitimate ascendants, the surviving spouse shall have a right to one-fourth of the hereditary

I press toward BAR OPERATIONS 2013 the mark for the

prize of the high calling of God in Christ Jesus. - Philippians 3:14

BARRISTERS’ CLUB

Page 6 of 67

in CIVIL LAW Prepared by: Atty. Roney Jone P. Gandeza property to Bubba. Phil later sells his undivided share to Matt. Is bubba entitled to redeem Phil’s share? Yes, because the right of legal redemption is not limited to the original co-owners. What matters is that the redemptioner (Bubba) was already a co-owner at the time when another co-owner (Phil) sells his undivided share. In the instant case, Bubba was already a co-owner when Phil sold his share. 22. Suppose Phil, instead of a sale, donated his undivided share to Matt, may Bubba or Rory, or both redeem? No, because the right of legal redemption may be exercised only if the share of a co-owner is alienated to a third person by onerous title. 23. Suppose Rory later sells his share to Bubba, may Matt redeem? No, because the right of legal redemption may be exercised only if the share of a co-owner is alienated to a third person by onerous title. The rationale behind the right of legal redemption among coowners is to reduce the number of co-owners and avoid entry of strangers into the co-ownership. If the alienation is in favor of a coowner, the number of co-owners is already reduced and no stranger has entered the co-ownership. QUESTIONS 24-25 are based on the following fact situation: Kobe, Lebron and Dwyane are co-owners of a parcel of land. Without the knowledge of his co-owners, Kobe sells the entire community property to Kevin. 24. Is the sale valid? A co-owner who sells the whole community property will affect

only his own share, and the transferee gets only what would correspond to his grantor in the partition of the thing owned in common. By virtue of the sale of the entire community property by a coowner, the buyer thereby becomes a co-owner of the property to the extent of participation of the seller co-owner since the sale produced the effect of substituting the buyer in the enjoyment thereof. In the instant case, Kevin thus acquired only the rights pertaining to the seller, Kobe, which is only onethird undivided share of the coowned lot. 25. Suppose Kobe sells the entire property to Kevin for P600, 000, what would be the basis of the redemption price? The redemption price should be one-third of the actual purchase price, (P200, 000) not of the prevailing market value of the property. Thus, Article 1088 of the Civil Code provides that “should any of the heirs sell his hereditary rights to a stranger before the partition, any or all of the co-heirs may be subrogated to the rights of the purchaser by reimbursing him of the price of the sale, provided they do so within the period of one month from the time they were notified in writing of the sale by the vendor. 26. Testator T died with a will. He left no compulsory heirs and, consequently, was completely free to dispose of his properties, without regard to legitimes. One of the clauses in his will perpetually prohibits the alienation or mortgage by his named legatees or devisees of the properties specified therein. Is the clause valid?

I press toward BAR OPERATIONS 2013 the mark for the

prize of the high calling of God in Christ Jesus. - Philippians 3:14

BARRISTERS’ CLUB

Page 7 of 67

in CIVIL LAW Prepared by: Atty. Roney Jone P. Gandeza No. The perpetual prohibition is valid for only 20 years. The testamentary provision stipulated in T’s will prohibiting perpetual alienation of mortgage of the properties mentioned therein violated Articles 867 and 870 of the Civil Code. Paragraph 4, Article 1013 of the same code which specifically allows a perpetual trust in inapplicable. Article 1013 is among the Civil Code provisions on intestate succession, specifically on the State inheriting from a decedent, in default of persons entitled to succeed. Under this article, the allowance for a permanent trust, approved by a court of law, covers property inherited by the State by virtue of intestate succession. The article does not cure a void testamentary provision which did not institute an heir. In the instant case, T’s estate cannot be subjected indefinitely to a trust because the ownership thereof would then effectively remain with him even in the afterlife. (Orendain v. Estate of Rodriguez, June 30, 2009)

diligence in the selection and supervision of his employees to avoid liability?

QUESTIONS 27-30 based on the following situation:

30. May damages owners?

No, he cannot. This is so because his liability as a common carrier is based on breach of contract. Such a defense will only serve to mitigate A’s liability because by then he will be considered as a debtor in good faith. 28. May B, owner of the cargo truck, invoke the same defense? Yes, because his liability is based on a quasi-delict. 29. May damages owners?

the bus invoke

claim both

moral vehicle

C can claim moral damages against B, owner of the cargo truck, because of the injuries he suffered, but as against A, owner of the bus, C can claim moral damages only if he proves reckless negligence on the part of the common carrier amounting to fraud.

are fact

A passenger bus owned by A and a cargo truck owned by B collided. C, a bus passenger, suffered injuries, while D, another bus passenger, died. The drivers of the two vehicles were at fault. C, the injured bus passenger, and E, the surviving heir of D, immediately filed suit against the owners of both vehicles for damages. 27. May A, successfully

C from

E from

claim both

moral vehicle

E can claim moral damages against both vehicle owners because the rules on damages arising from death due to a quasi-delict are also applicable to death of a passenger caused by breach of contract by a common carrier. (Arts. 1755, 1764, 2206 and 2219, CC) 31. To satisfy a final judgment rendered against H and W, husband and wife, the sheriff levied on a house and lot constituted by the couple as

owner, due

I press toward BAR OPERATIONS 2013 the mark for the

prize of the high calling of God in Christ Jesus. - Philippians 3:14

BARRISTERS’ CLUB

Page 8 of 67

in CIVIL LAW Prepared by: Atty. Roney Jone P. Gandeza their family home. The sheriff later sold the property at public auction to C, the judgment creditor.

32. What is an easement of aqueduct? Does the existence of an easement of right of way necessarily include the easement of aqueduct?

After some discussion, C agreed to allow the couple to remain on the property as lessees and to vacate the same in two years. A year later, the couple brought an action to annul the auction sale on the allegation that the property is their family home and its sale is void ab initio.

An easement of aqueduct is the right to make water flow thru intervening estates in order that one may make use of said waters. The existence of the easement of right of way does not necessarily include the easement of aqueduct. Consequently, the right to dig trenches and to lay pipelines for the conducting of water is not included in a contract granting a right of way (the rights given merely those of ingress and egress to and from the lot involved).

Will the action prosper? The action will not prosper. Even if the subject property is a family home and, thus, should have been exempt from execution, H and W should have asserted it as a family home and its being exempt from execution at the time it was levied upon or within reasonable time thereafter. Failure to do so would estop the spouses from later claiming the exemption.

33. X promised to donate his friend, Y, a parcel of land. Relying on such promise, Y constructed a house of strong materials on the land. When X died, however, his son Z inherited the land. In the suit filed by Z to recover possession of the land from Y, the latter invoked the right to be reimbursed of his necessary and useful improvements on the allegation that he is a builder in good faith.

It is evident that H and W did not assert their claim of exemption within a reasonable time. At no other time can the status of a residential house as a family home be set up and proved and its exemption from execution claimed but before the sale thereof at public auction.

Is Y a builder in good faith?

Settled is the rule that the right of exemption is a personal privilege granted to the judgment debtor and as such, it must be claimed not by the sheriff but by the debtor himself at the time of the levy or within a reasonable time thereafter. (De Mesa v. Acero, G.R. No. 185064, January 16, 2012)

No. The mere promise of X to donate the property to Y does not make the latter a builder in good faith. This is because at the time when the improvement was built, such promise was not yet fulfilled. It was therefore a mere possession by tolerance.

I press toward BAR OPERATIONS 2013 the mark for the

prize of the high calling of God in Christ Jesus. - Philippians 3:14

BARRISTERS’ CLUB

Page 9 of 67

in CIVIL LAW Prepared by: Atty. Roney Jone P. Gandeza A person whose occupation of a realty by sheer tolerance of its owner is not a possessor in good faith; hence, not entitled to the value of the improvements built thereon. (Verno Padua-Hilario v. Court of Appeals, et.al., January 19, 2000) 34. S contracts to sell to B a parcel of land. They agree that B shall pay the purchase price on October 25, and that in case of B’s failure to pay, the contract shall be automatically rescinded. If B does not pay on October 25, can he still pay on October 29? Yes, provided there has been no judicial or notarial demand for rescission of the contract as of October 29. Under Article 1592 of the Civil Code, in the sale of an immovable property, even though it may have been stipulated that upon failure to pay the price at the time agreed upon the rescission of the contract shall of right take place, the vendee may pay even after expiration of the period, as long as no demand for rescission of the contract has been made upon him either judicially or by a notarial act. 35. In a contract to sell, is it necessary for the vendor to send a notarial rescission when the vendee fails to pay the balance of the purchase price? No. Rescission, whether judicially or by notarial act, is not required to be done by the vendor. There can be no rescission of an obligation that is still non-existing, the suspensive condition not having happened. In a contract to sell, there is no contract to rescind, judicially or

by a notarial act, because from the moment the vendee fails to pay on time the purchase price, the contract between the parties is deemed ipso facto rescinded. 36. In a contract of sell, is the vendor obliged to refund what the vendee paid under the contract if the sale is not consummated? Yes. The partial payment made by the buyer must be returned to him, in the absence of a stipulation regarding forfeiture of payments made by the parties. Such action is but just and equitable under the premises. If it were otherwise, there will be unjust enrichment on the part of the seller at the expenses of the buyer. (Ordenv. Aurea, G.R. No. 172733 August 20, 2008) 37. When a co-owner seeks to exercise his right of legal redemption, within what period may he exercise such right? The right of legal redemption (or pre-emption) shall be exercised by a co-owner within 30 days from written notice by the seller-co-owner. Actual knowledge notwithstanding, written notice is still required. EXCEPTION: Actual knowledge by co-heirs or co-owners living in the same land with purchaser; or coowner was middleman in sale to third party, the requirement of written notice is dispensed with; period of redemption begins to run from actual knowledge. QUESTIONS 37-39 based on the following situation:

I press toward BAR OPERATIONS 2013 the mark for the

prize of the high calling of God in Christ Jesus. - Philippians 3:14

BARRISTERS’ CLUB

Page 10 of 67

are fact

in CIVIL LAW Prepared by: Atty. Roney Jone P. Gandeza A sold an unregistered land to B. Upon A’s death, his brother C, executed an extrajudicial settlement of A’s estate under which he (C) adjudicated exclusively unto himself the lot and simultaneously sold the same to D who was unaware of the prior sale. 37. Is the rule on double sale under Article 1544 of the Civil Code applicable to the above problem? No. Article 1544 if the Civil Code on double sales applies only where the same thing is sold to different vendees by the same vendor. It does not apply where the same thing is sold to different vendees by different vendors as in the case at bar. 38. Who between the two buyers is the rightful owner of the lot? B is the rightful owner of the lot. When A sold to B the property, ownership thereof was transferred to B in accordance with Article 1496 of the Civil Code which provides that the ownership of the thing sold is acquired by the vendee from the moment it is delivered to him in any of the ways specified in Articles 1497 to 1501. Article 1498, in turn, provides that when the sale is made through a public instrument, the execution thereof shall be equivalent to the delivery of the thing which is the object of the contract, if from the deed the contrary does not appear or cannot be clearly inferred. In the

problem presented, the sale between A and B contains nothing contrary to intent to transfer ownership. When A died in 1993, she no longer owned the lot, and, therefore, his brother could not have inherited it. The Extrajudicial Settlement with Simultaneous Sale did not thus confer upon C ownership of the lot in question; hence he could not have conveyed it to D. 39. Is the issue of D’s good faith relevant in solving whether he has a preferential right to the lot? The issue of good faith or bad faith is relevant only where the subject of the sale is a registered land and the purchaser is buying the same from the registered owner whose title to the land is clean. In such case the purchaser who relies on the clean title of the registered owner is protected if he is a purchaser in good faith and fro value. (Ong v. Olasiman, 465 SCRA 464 [2006]) 40. Article 40 of the Family Code specifically provides that the absolute nullity of a previous marriage may be invoked for purposes of remarriage on the basis solely of a final judgment declaring such previous marriage void. What are the consequences of this provision? a) Parties to a marriage are not permitted to judge for themselves the nullity of their marriage. Only the competent courts have that authority. b) Prior to such declaration, the validity of the first

I press toward BAR OPERATIONS 2013 the mark for the

prize of the high calling of God in Christ Jesus. - Philippians 3:14

BARRISTERS’ CLUB

Page 11 of 67

in CIVIL LAW Prepared by: Atty. Roney Jone P. Gandeza marriage question.

is

beyond

c) A party who contracts a second marriage assumes the risk of being prosecuted for bigamy. d) If the first marriage is void but a party remarries without seeking judicial nullity of his first marriage, the second marriage is also void for non-compliance with Article 40 in relation to Articles 51 to 53 of the Family Code. e) Bigamy is committed as long as the first marriage, although has not been declared as such under Article 40. (Mercado v. Tan, 337 SCRA 122) f) Bigamy is committed even if the second marriage is declared void on the ground of psychological incapacity of one of the spouses because such second marriage, although void, still produces legal consequences, among which is incurring criminal liability for bigamy. (Tenebro v. Court of Appeals, 424 SCRA 272) g) If the second marriage took place before the Family Code, Article 40 does not apply. Consequently, there is no need for a judicial decree of nullity of the first marriage. (Ty v. Court of Appeals, 346 SCRA 86)

h) If the second marriage took place after the Family Code, even if the first marriage took place before it, Article 40 applies because it has retroactive effect. (Atienza v. J. Brillantes, A.m. No. MTJ-92706, 243 SCRA 32) 41. S orally offers to sell his land to B for 300, 000. B accepts the offer and pays s the purchase price. S in turn delivers to B the title of the land. B now seeks to register the land in his name but the Register of Deeds refuses to register the sale in the absence of a notarized deed of sale. Which of the accurately describes remedy?

a) He may enter into the possession of the land as a buyer in good faith. b) He cannot compel S to return the payment because the contract is unenforceable. c) He may compel S to execute a notarized deed of sale because the contract is valid and enforceable. d) He may sue S to return the purchase price because no one may enrich himself at the expense of another. (C) is B’s best available remedy. The notarized deed of sale is required demanded by B is neither for validity nor for enforceability of

I press toward BAR OPERATIONS 2013 the mark for the

prize of the high calling of God in Christ Jesus. - Philippians 3:14

BARRISTERS’ CLUB

following B’s best

Page 12 of 67

in CIVIL LAW Prepared by: Atty. Roney Jone P. Gandeza the contract of sale. The sale is both valid and enforceable. The subject document is required by B only for his convenience; i.e., to allow him to register the sale. 42. Article 213 of the Family Code enunciates the rule that no child below the age of seven years shall be separated from the mother, except for compelling reasons. In custody cases, when should this provision be applied by the courts – at the time of the filing of the petition for custody or at the time when the court is to decide who between the parents is entitled to the custody of the child? The argument that the 7-year reference in the law applied to the date when the custody case is filed, not the date when the decision is rendered, is flawed. The matter of custody is permanent and unalterable. If the parent who was given custody (either by law or by choice of the child) suffers a future character change and becomes unfit, the matter of custody can always be re-examined and adjusted. (Espiritu, et.al., v. Court of Appeals., 242 SCRA 362 [1995]) QUESTIONS 43-44 based on the following situation:

are fact

X executes a real estate mortgage on his land in favor of Y to answer for any deficiency that may result from foreclosure of the chattel mortgage constituted over the car sold on installments by Y to Z.

43. Assuming there is a deficiency after foreclosure of the chattel mortgage, may Y foreclose the real estate mortgage constituted on X’s land? No, because in such a case, the third person-mortgagor (X), after paying the deficiency through foreclosure of the real estate mortgage, has the tight of a guarantor who can hold the vendee (z) liable for the payment made, thus indirectly violating the prohibition under the law. (Art, 1484 (3), Civil Code) 44. Suppose in the preceding problem Y assigns the promissory note to M, promising the latter that should Z default and the chattel mortgaged is foreclosed resulting in a deficiency, the assignor (Y) shall answer for the deficiency to the assignee (M). Is this stipulation valid? Yes, because in such a case, it is no longer the vendee (Z) who is held liable but the vendor (Y). There is this no violation of the Recto Law that if the vendor avails himself of the right to foreclose, he is prohibited from bringing an action against the purchase for the unpaid balance. 45. What are the instances under the law when form is an indispensable and mandatory requirement for the validity of the contract? Form is a mandatory requirement for the validity of the following contracts: (1) If the value of the personal property donated exceeds

I press toward BAR OPERATIONS 2013 the mark for the

prize of the high calling of God in Christ Jesus. - Philippians 3:14

BARRISTERS’ CLUB

Page 13 of 67

in CIVIL LAW Prepared by: Atty. Roney Jone P. Gandeza P5, 000, the donation and the acceptance shall be made in writing: otherwise, the donation is void. (par. 3, Art. 748, CC); (2) Donation of an immovable, regardless of value, must be in a public instrument (Art 748, CC); (3) A contract of partnership is void whenever property is contributed thereto, if an inventory of said property is not made, signed by the parties, and attached to the public instruments (Art. 1773, CC); (4) Sale of piece of through an agent 1847, CC);

land (Art.

(5) Antichresis (Art. 134, CC); and (6) Payment of interest (Art. 1956, CC). 46. If a marriage is declared void, how should the properties acquired by the spouses “during the marriage” be liquidated, partitioned, and distributed? If a marriage is declared void, the properties of the spouses should be owned in the concept of coownership. A void or voidable marriage, regardless of its ground, cannot be governed by the conjugal partnership of gains nor by the absolute community of property regime. In such a case, Article 147of the Family Code applies, except when the marriage is declared void for being bigamous, in which case Article 148 governs.

(RJPG: The above rules assumes significance in light of Articles 102 (absolute community or property) and 129 (conjugal partnership) of the Family Code which commonly provide that the conjugal dwelling shall be adjudicated to the spouse with whom the majority of the common children choose to remain. Applying the above rule, if a marriage is declared void, the conjugal dwelling shall be partitioned not in accordance with Articles 102 and 129, but in accordance with Articles 147 and 148 of the Family Code. This holds true even if majority of the children choose to remain with one parent.) 47. In an action for annulment of marriage, who are the parties who ay commence the action and within what period may the action be filed? The following are the parties who may commence an action for annulment of marriage and the periods for the filing of such action: a) LACK OF PARENTAL CONSENT: The minor should bring the action within five years after attaining the age of 21. For the parent or guardian, the action must be brought at any time before such party reaches the age of 21. b) INSANITY: The sane spouse or person having legal charge of the insane spouse must bring the action at any time before the death of either party. The insane spouse must bring the action during a lucid interval or after regaining sanity, also before the death of the other party. The reason in not providing for a 5-year period is the insanity recurs.

I press toward BAR OPERATIONS 2013 the mark for the

prize of the high calling of God in Christ Jesus. - Philippians 3:14

BARRISTERS’ CLUB

Page 14 of 67

in CIVIL LAW Prepared by: Atty. Roney Jone P. Gandeza c) FRAUD: Injured party must bring the action within five years after discovery of the fraud. d) FORCE, INTIMIDATION, UNDUE INFLUENCE: The injured party must bring the action within five years from the time the force, intimidation or undue influence disappeared or ceased. e) PHYSICAL INCAPACITY: The injured party must bring the action within five years after the marriage. f) SEXUALLY-TRANSMISSIBLE DISEASE: The injured party must bring the action within five years after the marriage. 48. Under the Domestic Adoption Act, may the adopting parent file for rescission of the decree of adoption?

wedding. A month following Debby’s birth, Cora died in a car accident. Andy reunited with Betty and immediately married her. Is Debby legitimated by the marriage of her parents? Yes. It is clear from the problem that although Debby was conceived and born outside of wedlock of her parents, she was conceived at the time when her parents, Andy and Betty, were not disqualified by any impediment to marry each other. The reckoning point for legitimation is the period of conception the child, not his birth, and the subsequent valid marriage of the parents. 49. What is the new interpretation given to the second paragraph of Article 26 of the Family Code?

No. The Domestic Adoption Act had already abrogated the right of the adopting parent to rescind a decree of adoption. Nevertheless, the adopting parent can always, for valid reasons, cause forfeiture of certain benefits that would otherwise accrue to the adopted. For instance, upon the grounds provided by law, an adopter may deny the adopted his legitime and by a will, may exclude him from having a share in the disposable free portion. (Lahom v. Sibulo, GR No. 143989, July 14, 2003)

In Republic v. Orbecido III, 472 SCRA 114 [2005], the Supreme Court made a novel pronouncement that paragraph 2 of Article 26 of the Family Code should be interpreted to include cases involving parties who, at the time of the marriage, were Filipino citizens but later one of them becomes a naturalized citizen of a foreign country and obtains a divorce decree. The Filipino spouse should likewise be allowed to remarry as if the other party were a foreigner at the time of the marriage. To rule otherwise will be a sanction absurdity and injustice.

49. Andy and Betty, both eligible to marry each other, cohabited as husband and wife without the benefit of marriage. A few weeks before Betty was to give birth, however, Andy married Cora, an old maid. Heartbroken, Betty gave birth to Debby a few weeks after Andy’s

ANOTHER ANSWER: As pointed out by the Supreme Court in Obrecido, the reckoning point is not the citizenship of the parties at the time of the celebration of the marriage, but their citizenship at the time a valid divorce is obtained

I press toward BAR OPERATIONS 2013 the mark for the

prize of the high calling of God in Christ Jesus. - Philippians 3:14

BARRISTERS’ CLUB

Page 15 of 67

in CIVIL LAW Prepared by: Atty. Roney Jone P. Gandeza abroad by alien spouse capacitating the latter to remarry. QUESTION 50-51 are based on the following fact situation: William, an American, and Marissa, a Filipina, cohabited as husband and wife without the benefit of marriage. During their cohabitation, the couple bought from Mauricio a parcel of land in Baguio City. Although the deed of sale was placed in the names of both William and Marissa as buyers, the sale was registered in the name of Marissa alone because William was disqualified to own real properties in the Philippines. It is sufficiently established that the funds used to buy the property came solely from William, as Marissa has no sufficient source of income. After their relationship has turned sour and the two went separate ways, William sold all his rights and interests in the property to Nicasio, a Filipino. When Nicasio tried to register the property in his name, he discovered that the certificate of title is already registered in the name of Marissa, and that it has already been mortgaged. 50. If William is the true buyer of the property, what is the effect of the registration of the property in the name of Marissa? The registration of the property in the name of Marissa does not make her the owner of the property in question. It is settled that registration is not a mode of acquiring ownership. It only means of confirming the fact of

its existence with notice to the whole world at large. Certificates of title are not a source of right. The mere possession of a certificate of title does not make the holder the true owner of the property. Thus, the mere fact that Marissa has the title of the disputed property in her name does not necessarily, conclusively, and absolutely make her the owner. 51. Given that William is disqualified to own real properties in the Philippines, is the sale between him and Nicasio valid? Given that aliens are disqualified to own real properties in the Philippines, the transfer of the subject property from Mauricio to William, who is an American, would have been declared invalid if challenged, had not William conveyed the property to Nicasio who is a Filipino citizen. It has been the consistent ruling of the Supreme Court that if the land is invalidly transferred to an alien, who subsequently becomes a Filipino citizen, or transfers it to a Filipino, the flaw in the original transaction is considered cured and the title of the transferee is rendered valid. (Borromeo v. Descallar, GR No. 159310, Febraury 24, 2009) 52. John, an American, and Nieves, a Filipina, were married in Cebu City in 1997. During their marriage, Nieves bought from Dionisia a beach lot in Boracay for P1.2 million. The sale was financed by John. The couple, also using John’s funds, introduced improvement on the beach lot and eventually converted the property into a vacation and tourist resort.

I press toward BAR OPERATIONS 2013 the mark for the

prize of the high calling of God in Christ Jesus. - Philippians 3:14

BARRISTERS’ CLUB

Page 16 of 67

in CIVIL LAW Prepared by: Atty. Roney Jone P. Gandeza When the couple had a failing out, Nieves leased the property to Matthews for a period of 25 years. She did so without John’s consent. Does John have the legal standing to question the validity of the lease agreement on the theory that in so doing, he was merely exercising his prerogative as a husband regarding conjugal property? John, being an alien, is absolutely prohibited from acquiring public and private lands in the Philippines. Considering that Nieves appeared to be the designated vendee of the property leased, she acquired sole ownership thereto. This is true even of John’s claim that he provided the funds for such acquisition is to be sustained. By entering into such contract knowing that it was illegal, no implied trust was created in his favor; no reimbursement for his expenses can be allowed; and no declaration can be made that the subject property was part of the conjugal or community property of the spouses. In any event, he had and has no capacity or personality to question the subsequent lease of the property by his wife in the theory that in so doing, he was merely exercising the prerogative of a husband in respect of conjugal property. To sustain such a theory would countenance indirect violation of the constitutional prohibition. If the property were to be declared conjugal, as he would then have a decisive vote as to its transfer or disposition. This is a right that the Constitution does not permit him to have. Thus, the validity of the lease

must be upheld. (Mathews v. Taylor, June 22, 2009) 53. What are the recognized exceptions under the Constitution allowing aliens to own private lands in the Philippines? The instances when aliens are allowed to acquire private lands in the Philippines are: (a) By hereditary succession (Section 7, Article XII, 1987 Constitution); (b) A natural-born citizen of the Philippines who has lost his Philippine citizenship may become a transferee of private lands, subject to the limitations provided by law. (Section 8, Article XII, Constitution). (Republic Act No. 8179 now allows a former natural-born Filipino citizen to acquire up to 5,000 square meter of urban land and 3 hectares of rural land, and he may now use the land not only for residential purposes, but even for business or other purpose); and (c) Americans who may have acquired title to private lands during the effectivity of the Parity Agreement shall hold valid title thereto as against private persons (Section 11, Article XII, 1973 Constitution). 54. Mao Tse Tung, a Chinese national, bought from Juan dela Cruz, a Filipino, a parcel of land in Bulacan. Upon Mao’s death, the property was transferred by succession to his son, Chiang Kai Shek. When Chiang died, the property was in turn transferred by succession to his Filipino wife, Maria. May successfully

the government prosecute a

I press toward BAR OPERATIONS 2013 the mark for the

prize of the high calling of God in Christ Jesus. - Philippians 3:14

BARRISTERS’ CLUB

Page 17 of 67

in CIVIL LAW Prepared by: Atty. Roney Jone P. Gandeza reversion case on the argument that the sale of the land to Mao violated the Constitutional provision disallowing aliens from acquiring public and private lands in the Philippines? If the government had commenced reversion proceedings when the lot was still in the hands of Mao Tse Tung who was an alien disqualified to hold title thereto, the reversion of the land to the State would undoubtedly be allowed. However, this is not the case here. When the government instituted the reversion case, the lot had already been transferred by succession to Maria who is a Filipino citizen. And since the lot was transferred to a Filipino citizen, the flaw in the original transaction is considered cured. (Republic v. Register of Deeds, GR 158230, July 16, 2008)

brother Nicanor and their father Vicente. Is this argument tenable? The contention that an implied trust was created between Nicanor and his father does not hold water because the prohibition against an alien from owning lands of the public domain is absolute and not even an implied trust can be permitted to arise on equity considerations. In the instant case, Nicanor became the owner of the subject when he was granted a miscellaneous sales patent by the Bureau of Lands. Under the law, a certificate of title issued pursuant to any grant or patent involving public land is a conclusive and indefeasible as any other certificate of title issued to private lands in the ordinary or cadastral registration proceeding.

55. Vicente Ting, a Chinese national married to a Filipina, occupied a public land in Olongapo City. Pursuant to an affidavit, Vicente transferred, without valuable consideration, all his rights and interests over the lot in favor of his eldest son, Nicanor. On the basis of the affidavit, Nicanor, who earlier obtained Filipino citizenship, was issued a miscellaneous sales patent by the Bureau of Lands. Not long after, the Register of Deeds issued an original certificate of title over the property in the name of Nicanor.

The effect of the registration of a patent and the issuance of a certificate of title to the patentee is to vest in him an incontestable title to the land, in the same manner as if ownership had been determined by final decree of the court, and the title so issued is absolutely conclusive and indisputable, and is not subject to collateral attack, consequently, the lot is not part of Vicente Ting’s estate. (Ting Ho, Jr. v. Teng Gui, GR No. 130115, July 16, 2008)

Upon Vicente’s death, Nicanor’s brothers and sisters filed for judicial partition of the lot on the allegation that an implied trust over said property was created between their

Marcel and Monique, French nationals, entered into a Contract to Sell with Fil-Estate Realty Corp. for the purchase of a residential unit in a townhouse project. When Fil-Estate failed to

QUESTIONS 56-57 based on the following situation:

I press toward BAR OPERATIONS 2013 the mark for the

prize of the high calling of God in Christ Jesus. - Philippians 3:14

BARRISTERS’ CLUB

Page 18 of 67

are fact

in CIVIL LAW Prepared by: Atty. Roney Jone P. Gandeza comply with its verbal promise to complete the project at a certain date, the couple filed for rescission of the contract. 56. What it the status of the contract entered into by Marcel and Monique with Fil-Estate Realty Corp.? The Contract to Sell is void. Since Marcel and Monique, being French nationals, are prescribed under the Constitution from acquiring and owning real property, it follows that the Contract to Sell entered into by the parties is void. Under the Civil Code, all contracts whose cause, object or purpose is contrary to law or public policy and those expressly prohibited or declared void by law are inexistent and void from the beginning. A void contract is equivalent to nothing; it produces no civil effect. 57. In any event, are Marcel and Monique entitled to recover from Fil-Estate the amount paid as well as interest and other damages? Since the contract involved here is a Contract to Sell, ownership has not yet transferred to Marcel and Monique when they filed the suit for rescission. While the intent to circumvent the constitutional prescription on aliens owning real property was evident by virtue of the execution of the Contract to Sell, such violation if the law did not materialize because the spouses caused the rescission of the contract before the execution of the final deed transferring ownership. Thus, the exception to the application of the pari delicto doctrine

finds application in the case at bar. Under the law, one who repudiates the agreement and demands his money before the illegal act has taken place is entitled to recover. Marcel and Monique are therefore entitled to the recovery only of the purchase price. No damages may be recovered on the basis of a void contract; being nonexistent, the agreement produces no juridical tie between the parties involved. Further, the spouses are not entitled to actual as well as interest thereon, moral and exemplary damages and attorney’ fees. (Hulst v. PR Builders, Inc., 532 SCRA 41 [2007]) 58. H and W were married in 1978. In 1992, H obtained a decree of legal separation after catching his wife having illicit relations with their neighbor. In the final decree of legal separation issued by the court, the court ordered the forfeiture of W’s share in the net profits earned by the conjugal partnership in favor of her children pursuant to Article 63(2) in relation to Article 129 of the Family Code. Not satisfied with the ruling regarding the forfeiture of her share in the conjugal assets, W claims that the net assets of the conjugal partnership shall be computed in accordance with Article 102 of the Family Code (a provision under absolute community of property), instead of Article 129 (a provision on conjugal partnership). She argues that Article 102 applies because there are no other provisions under the Family Code which defines net profits subject of forfeiture as a result of legal

I press toward BAR OPERATIONS 2013 the mark for the

prize of the high calling of God in Christ Jesus. - Philippians 3:14

BARRISTERS’ CLUB

Page 19 of 67

in CIVIL LAW Prepared by: Atty. Roney Jone P. Gandeza separation. She contends that her vested right over half of the common properties of the conjugal partnership is violated when the forfeiture is to be made pursuant to Article 129 in relation to Article 63(2) of the Family Code. W’s move following issues:

raises

the

a) What law governs the property relations of the spouses given that they were married before the effectivity of the Family Code? b) What law governs the dissolution of their common properties since the decree of legal separation was issued after the Family Code is already in effect? As to their property relations, the Spouses are governed by the regime of conjugal partnership of gains. This is so because they were married when the operative law was the Civil Code. As to the liquidation of their conjugal partnership assets, however, the Family Code is applicable because it is already the operative law at the time of the dissolution of their conjugal partnership. In the instant case, the applicable law in so far as the liquidation of the conjugal partnership assets and liabilities of H and W is concerned is Article 129 of the Family Code (liquidation of the conjugal partnership) in relation to Article 63 (effects of a decree of legal separation). The latter provision is applicable because insofar as Article

256 of the Family Code “[t]his code shall have retroactive effect insofar as it does not prejudice or impair vested or acquired rights in accordance with the Civil Code or other law.” W’s contention that her vested right over half of the common properties of the conjugal partnership is violated when her share in the conjugal partnership is forfeited in favor of her children pursuant to Article 63(2) and 129 of the Family Code has no basis. While it is true that the couple were married at the time when the operative law was the Civil Code, the Family Code should be given retroactive application for purposes of determining the “net profits earned” by the conjugal partnership which is subject to forfeiture. A spouse’s claim of a vested right is not etched in stone. To be vested, a right must have become a title – legal or equitable – to the present or future enjoyment of property. In one case, the Supreme Court reiterated its long standing ruling that “prior to the liquidation of the conjugal partnership, the interest of each spouse in the conjugal assets is inchoate, a mere expectancy, which constitutes neither a legal nor an equitable estate, and does not ripen into title until it appears that there are assets in the community as a result of the liquidation and settlement. The interest of each spouse is limited to the net remainder resulting from the liquidation of the affairs of the partnership after its dissolution. Thus, the right of the husband or wife to one-half of the conjugal assets does not vest until the dissolution of the marriage, when it is finally determined that, after settlement of conjugal obligations,

I press toward BAR OPERATIONS 2013 the mark for the

prize of the high calling of God in Christ Jesus. - Philippians 3:14

BARRISTERS’ CLUB

Page 20 of 67

in CIVIL LAW Prepared by: Atty. Roney Jone P. Gandeza there are net assets left which can be divided between the spouses or their respective heirs.” 59. Is the computation of net profits earned in the conjugal partnership of gains the same as the computation of net profits earned in the absolute community? The term net profits is defined in Article 102(4) of the Family Code. Under this provision, the term net profits “shall be the increase in value between the market value of the community property at the time of the celebration of the marriage and the market value at the time of its dissolution.” Without any doubt, Article 102(4) applies to both the dissolution of the absolute community regime under Article 102 of the Family Code, and to the dissolution of the conjugal partnership regime under Article 129 of the Family Code. The difference lies in the process used under the dissolution of the absolute community regime under Article 102 of the Family Code, and in the processes used under the dissolution of the conjugal partnership regime under Article 129 of the Family Code. ON ABSOLUTE COMMUNITY REGIME: Applying Article 102 of the Family Code, the “net profits” requires a prior determination of the market value of the properties at the time of the community’s dissolution. From the totality of the market value of all the properties, the debts and obligations of the absolute community are to be deducted and this will result to the net assets or net remainder of the properties of the absolute community, from which the value of the properties at the time of

marriage is to be deducted, which then results to the net profits. ON CONJUGAL PARTNERSHIP REGIME: Applying Article 129 of the Family Code, the “net profits” requires a prior determination of the separate properties and debts of the spouses under the following procedure a) an inventory shall be prepared, listing separately all the properties of the conjugal partnership and the exclusive properties of each spouse; b) amounts advanced by the conjugal partnership in payments of personal debts and obligations of either spouse shall be credited to the conjugal partnership as an asset thereof; c) each spouse shall be reimbursed for the use of his or her exclusive funds in the acquisition of property or for the value of his or her exclusive property, the ownership of which has been vested by law in the conjugal partnership; d) the debts and obligations of the conjugal partnership shall be paid out of the conjugal assets. In case of insufficiency of said assets, the spouses shall be solidarily liable for the unpaid balance with their separate properties, in accordance with the provisions of paragraph 2 of Article 121; e) whatever remains of the exclusive properties of the spouses shall thereafter be delivered to each of them; f) unless the owner had been indemnified from whatever source, the loss or deterioration of movables used for the benefit of the family, belonging to either spouse, even due to fortuitous event, shall be paid to said spouse from the conjugal funds, if any; and g) the net remainder of the conjugal partnership shall constitute the profits, which shall be divided equally between husband and wife, unless a different

I press toward BAR OPERATIONS 2013 the mark for the

prize of the high calling of God in Christ Jesus. - Philippians 3:14

BARRISTERS’ CLUB

Page 21 of 67

in CIVIL LAW Prepared by: Atty. Roney Jone P. Gandeza proportion or division was agreed upon in the marriage settlements or unless there has been a voluntary waiver or forfeiture of such share as provided in the Family Code. 60. Suppose Article 102 of the Family Code (which is a provision under the regime of absolute community of property) is to apply in the instant case, is W entitled to receive anything from the absolute community? If H and W have no separate properties, the remaining properties of the couple are all part of the absolute community. And its market value at the time of the dissolution of the absolute community constitutes the “market value at dissolution.” When H and W were legally separated, all the properties which remained will be liable for the debts and obligations of the community. Such debts and obligations will be subtracted from the “market value at dissolution.” What remains after the debts and obligations have been paid from the total assets of the absolute community constitutes the net remainder or net asset. And from such net asset or net remainder off the couple’s remaining properties, the market value at the time of the marriage will be subtracted and the resulting totality constitutes the “net profits.” Since both H and W have no separate properties, and nothing would be returned to each of them, what will be divided equally between them are simply the net profits. However, the trial court forfeited the half-share of W in favor of her children. Thus, if Article 102 is used in the instant case (which should not be the case), nothing is left to W since both parties entered into their

marriage without bringing with them any property. 61. Given that Article 129 of the Family Code applies to the liquidation of the conjugal assets of H and W, is the latter entitled to receive any property from the conjugal partnership? No. What remains in the conjugal properties of H and W (after payment of all debts and obligations) should be divided equally between the spouses. However, since W herself is the guilty party, her share from the “net profits” of the conjugal partnership is forfeited in favor of the common children pursuant to Article 63(2) of the Family Code. Nothing will be returned to W because in the conjugal partnership regime, there is no separate property which may be accounted for in the guilty party’s favor. (Quiao v. Quiao G.R. No. 176556, July 4, 2012) 62. S sells to B a retro a house and lot for P2.5 million. The agreement of sale provides that S cannot repurchase the property within three years from the date of the contract. If the sale is entered into in 2001, when can S repurchase the property? S can repurchase the property within four years from the expiration of the time within which the right to redeem cannot be exercised (here three years from date of the contract). In other words, S can repurchase the property until 2008 which is four years from 2004, the date of the expiration of the time within which his right to redeem cannot be exercised. This is pursuant to paragraph 1 of Article 1606 of the Civil Code which provides that the

I press toward BAR OPERATIONS 2013 the mark for the

prize of the high calling of God in Christ Jesus. - Philippians 3:14

BARRISTERS’ CLUB

Page 22 of 67

in CIVIL LAW Prepared by: Atty. Roney Jone P. Gandeza right of the seller to repurchase the property sold, in the absence of an agreement as to the period of repurchase, shall last four years from the date of the contract. 63. Suppose it is stipulated in the agreement of sale that the vendor a retro can repurchase the property “whenever he or his heirs have the means,” when can S repurchase? S can repurchase the property within ten years from 2001; i.e., until 2010, if the parties agree on the right to redeem without specifying the period of redemption but from the situation, facts or circumstances, it can be inferred that the parties intended a period, the vendor a retro may deem within ten years from the date of the property sold within ten years from the date of the contract. 64. S offers to B in a letter the sale of a parcel of land. B sends a reply. Which of the following statements in B’s reply will not result in a contract? a) “I accept your offer to sell the land. I wish I could have gotten a better price.” b) “I accept your offer to sell the land, but can you shave the price?” c) “I accept your offer to sell the land, but only if I can pay on 90 days credit.” d) I accept your offer to sell the land, provided that you are the owner.”

shoes assent ot agreement to the offer. The acceptance must be unequivocal and communicated to the offeror. The acceptance in (c) is not an unequivocal acceptance because of the condition to pay on credit; such a condition operates as a counter-offer. An acceptance may be unequivocal even though the offeree expresses dissatisfaction with the offer, as in the case of (a) and (b). The condition in (d) that the offeror owns the property is implied in every sale of land, so the condition does not add any new or different terms to the offer. 65. A local civil registrar of a remote town in Ilocos Sur issued a marriage license on the same day that the applicants filed their application. Will it affect the validity of the marriage? No, it will not. The failure of the local civil registrar to comply with the publication requirement under Article 17 of the family Code is considered as a mere irregularity in a formal requisite of marriage which will not affect the validity of the marriage, but would subject the guilty party administratively, civilly or criminally liable. 66. Cindy offers to sell to Daria a particular car for P300, 000. Which of the following events will NOT terminate Cindy’s offer? a. A dies prior to B’s acceptance, and at the time B accepts, he is unaware of A’s death.

(C) is correct. Acceptance is a voluntary act by the offeree that

I press toward BAR OPERATIONS 2013 the mark for the

prize of the high calling of God in Christ Jesus. - Philippians 3:14

BARRISTERS’ CLUB

Page 23 of 67

in CIVIL LAW Prepared by: Atty. Roney Jone P. Gandeza b. The night before B accepts, fire destroys the car. c. B pays 1,000 for a 30-day option to buy the car. During this period, A dies, and later A accepts the offer, knowing of A’s death. d. A dies an hour before receiving B’s acceptance. (C) will not terminate Cindy’s offer. As a rule, the death of either the offeror or offeree terminates the offer, except when the offer is irrevocable as in the case of an offer founded upon an option. The event in (A) will terminate Cindy’s offer. The offeree’s power of acceptance is terminated when the offeror of offeree dies or is deprived of legal capacity to enter into the proposed contract. An offer is personal to both parties and cannot pass to the decedent’s heirs or assigns. The event in (B) will automatically terminate Cindy’s offer if the specific subject matter of the offer is destroyed before the offer is accepted. (D) will also terminate Cindy’s offer for the same reason as in (A). 67. In which of the following fact situations would a court most likely find that an implied contract existed? a) X, a noted licensed physician, sees an unconscious pedestrian lying bleeding on the shoulder of the highway. Z

stops his car, gets out, and renders emergency medical care to the injured pedestrian. b) X, 82, asks her favorite niece, Y, a CPA, to fill out and file his income tax return. X’s only income is from this monthly pension from SSS and interest income on a bank account. It takes Y five minutes to complete the form, takes a taxi, and goes to personally file the return at the BIR. c) X, a contractor, has a contract to paint Y’s house. X mistakes the house of Y’s neighbor, Z, for Y’s house. As X paints Z’s house, Z stand by watching until the job is done. d) A, a homeowner, has already paid his realty property tax. A clerk in the treasurer’s office mistakenly sends A a bill that should have gone to A’s neighbor, B. Being a good citizen and the thinking that the treasurer’s office would not have sent him the bill if he did not owe the money, A pays the bill. An implied contract existed in (C). An implied contract is a contract formed by manifestations of the parties other than oral or written language, i.e., by conduct. In this case, X, by beginning to paint Z’s house, has made an offer by his conduct because a reasonable person would conclude that the services were offered with the expectation of

I press toward BAR OPERATIONS 2013 the mark for the

prize of the high calling of God in Christ Jesus. - Philippians 3:14

BARRISTERS’ CLUB

Page 24 of 67

in CIVIL LAW Prepared by: Atty. Roney Jone P. Gandeza compensation rather than gratuitously. X’s offer has been accepted by Z even though the latter said nothing. The situation in (A) does not give rise to an implied contract because the pedestrian has not manifested his consent to the offer made by X (in contrast to a patient who goes to X’s office and submits to treatment by hi,). X is not without a remedy. He may recover the value of his services under a quasi-contract. A quasi-contract is not really a contract at all; rather, it is a legal fiction designed to avoid injustice by preventing unjust enrichment of one party to the detriment of another. The situation in (B) does not create an implied contract. The conduct of X and Y does not appear from an objective standard to manifest contractual intent because of the close family relationship of the parties and the minor burden on Y to render her services. Courts generally will not presume that a contractual relationship was intended under these circumstances. The situation in (D) likewise does not create an implied contract. Without some conduct on the part of B, such as knowingly accepting the offered benefits in silence, a court will not find the manifestation of mutual consent necessary for an implied contract. 68. A man, prior to his marriage, made a donation in a public instrument, in favor of his future wife, on condition that should she die before him and there be no children, one-half of the properties donated shall be given to the parents of his wife.

Nine months after the wedding, the wife died without issue. The parents now claim the one-half share given to them in the deed of donation. Are the parents entitled to the property? The parents cannot get the one-half share. Insofar as said share is concerned, it cannot be a valid donation propter nuptias nor a donation inter vivos nor a donation mortis causa. The donation is not a donation propter nuptias because the share was not given to one of the spouses. It is not a donation inter vivos, for there was no acceptance on the part of the parents. It is not a donation mortis causa because the deed of donation did not have the formalities of a will, aside from the fact that the donor is still alive. 69. Seller is obliged to deliver to Buyer one of his cars. Is the object of the sale determinate or indeterminate? The object of the sale refers to a class which in itself is determinate. Here, the particular thing to be delivered is DETERMINABLE without the need of a new contract between the parties (Art. 1349, CC); it becomes determinate upon delivery. 70. What is the “mirror doctrine” in the law on sales? A purchaser of property cannot close his eyes to facts which should put a reasonable man on his guard and claim that he acted in good faith under the belief that there was no defect on the vendor’s title.

I press toward BAR OPERATIONS 2013 the mark for the

prize of the high calling of God in Christ Jesus. - Philippians 3:14

BARRISTERS’ CLUB

Page 25 of 67

in CIVIL LAW Prepared by: Atty. Roney Jone P. Gandeza The refusal to believe in the strong possibility of a defect on the vendor’s title will not make the purchaser an innocent purchase for value, if circumstances are such that a reasonably prudent man would have taken the necessary precaution if in the same situation. (Embrado v. CA, 233 SCRA 333) 71. Apple Computers, Inc. and a Philippine distributor entered into an agreement whereby the distributor agreed to order 1,000 units of Apple Computers every month and to resell them in the Philippines at the manufacturer’s suggested price plus 10%. All unsold units at the end of the year shall be bought back by the manufacturer at the same price they were ordered. The manufacturer shall hold the distributor free and harmless from any claim for defects in the unit. Is the agreement one of sale or agency? The contract is one of agency, not one of sale. Sale is negated by the following circumstances: 

the price is fixed by the manufacturer with the 10% mark-up constituting the commission.



the manufacturer reacquires the unsold units at exactly the same price.



warranty for the units was borne by the manufacturer.

The foregoing circumstances indicate a sale because ownership of

the units was never intended to transfer to the distributor. 72. Francisco donated to Cirila a parcel of land. The donation was made in a public instrument, while the acceptance mad by Cirila was embodied in the same public instrument. Upon the death of Francisco, his nephews and nieces brought an action for the recovery of the property on the ground that the done, Cirila, was the common-law wife of their uncle at the time of the donation, and that it is therefore coid under Article 87 of the Family Code. Will the action prosper? Yes. Article 87 of the Family Code expressly provides that every donation or grant of gratuitous advantage, direct or indirect, between the spouses during the marriage shall be void, except moderate gifts which the spouses may give each other on the occasion of any family rejoicing. The prohibition shall also apply to persons living together as husband and wife without a valid marriage. Since Francisco and Cirilia lived together as husband and wife without valid marriage, the inescapable conclusion is that the donation made by Francisco in favor of Cirila is void under Article 87 of the Family Code. (Arcaba v. Batocael, 370 SCRA 414 [2001]) 73. B buys from S a car, payable in installments. As security for the payment of the balance of the purchase price, a chattel mortgage is constituted on the car in favor of S. As additional security for the payment of the balance, C, a friend of B, executes a real

I press toward BAR OPERATIONS 2013 the mark for the

prize of the high calling of God in Christ Jesus. - Philippians 3:14

BARRISTERS’ CLUB

Page 26 of 67

in CIVIL LAW Prepared by: Atty. Roney Jone P. Gandeza estate mortgage over his land in favor of S.

May B enforce against S the warranty against eviction?

Is the Recto Law applicable if additional security is given by a third person who executes a real estate mortgage to answer for a deficiency at foreclosure?

Yes. While it appears that B is evicted by virtue of a right of subsequent to the sale to him (not prior to the sale), the warranty may still be enforced because the cause of B’s eviction is imputable to the vendor S.

Yes, because in such a case, the third person-mortgagor (C), after paying the deficiency through foreclosure of the real estate mortgage, has the right of a guarantor who can hold the buyer (B) liable for the payment made, thus indirectly violating the Recto Law. The Recto Law prohibits foreclosure for such real estate mortgage because C would be entitled to proceed against B under the law on guaranty, thus resulting in indirect circumvention of the prohibition. 74. An agent is allowed to sell to himself what his principal has ordered him to buy. Is this statement accurate? That statement is inaccurate. The agent is allowed to sell to himself what his principal has ordered him to buy but only with the permission of his principal. If he does so without such permission, the sale is void. 75. S sells a piece of land to B who does not register the sale. Subsequently, S sells the same piece of land to C who registers the sale. In a suit between the buyers and B and C over the land sold, B (the first buyer) is defeated, and is deprived of the property.

76. A offers to sell to B a particular car for P300, 000. Which of the following events creates a contract between A and B? a) A dies prior to B’s acceptance, and at the time B accepts, he is unaware of A’s death. b) The night before B accepts, fire destroys the car. c) B pays 1,000 for a 30-day option to buy the car. During this period, A dies, and later B accepts the offer, knowing of A’s death. d) A dies an hour before receiving B’s acceptance. The event in (C) creates a contract between A and B because of the option money which continues to be effective despite the offeror’s death, and despite the offeree’s knowledge of such death. 77. S sold a retro to B a parcel of land. Within the period stipulated for redemption, S failed to redeem. To register in the Registry of Property his consolidation of ownership, B filed a petition for consolidation,

I press toward BAR OPERATIONS 2013 the mark for the

prize of the high calling of God in Christ Jesus. - Philippians 3:14

BARRISTERS’ CLUB

Page 27 of 67

in CIVIL LAW Prepared by: Atty. Roney Jone P. Gandeza but did not name S as respondent. Consequently, S was not duly summoned and heard. Has the jurisdiction?

court

contemplated by law. Such belated acceptance of Dr. Lim partakes of an offer by him to Dr. Go which the latter is free to accept or not.

acquired

No. S should have been named as a respondent in the case, should have been summoned, and should have been heard. The requirement under Article 1607 of the Civil Code for a “judicial order” is not required for consolidation of ownership of the property, but for purposes of registering the consolidation of title. 78. Dr. Go offered a partnership in his private practice to Dr. Lim, a young surgeon. Dr, Lim, hoping to secure a more favorable opportunity, declined Dr. Go’s offer. A few weeks later, Dr. Lim wrote to Dr. Go saying that he had reconsidered the matter and was now accepting the offer previously made. Did a contract arise? a) Yes, because Dr. Go’s offer had no termination date. b) Yes, because Dr. eventually accepted offer in writing.

Lim the

c) No, because Dr. Lim’s rejection terminated the offer. d) No, because Dr. Lim’s refusal constitutes a counter-offer. (C) is correct because Dr. Lim’s rejection terminated Dr. Go’s offer. Dr, Lim’s belated acceptance of the offer is not the acceptance of an offer

79. A friend offered to sell Henry a laptop for P10,000. Henry replied, saying, “I’ll pay you P6,000 now and give you the balance at the end of the month.” Did a contract arise? a) Yes, because Henry’s acceptance of the offer was unconditional. b) Yes, because the conditions added by Henry merely involves the manner of payment of the purchase price. c) No, because the breach of the obligation is substantial enough that would invalidate the consent of the offeror. d) No because the conditions of payment added by Henry constitutes a counter-offer, thus invalidating the original offer. (D) is correct because the conditions of payment added by Henry constitute a counter-offer, thus invalidating the original offer. 80. What are the steps to be undertaken by the creditor in case of non-payment of the debt on the things pledged? The steps are:

I press toward BAR OPERATIONS 2013 the mark for the

prize of the high calling of God in Christ Jesus. - Philippians 3:14

BARRISTERS’ CLUB

Page 28 of 67

in CIVIL LAW Prepared by: Atty. Roney Jone P. Gandeza (a) The creditor proceeds to a Notary Public for the sale of the thing pledged; (b) The sale shall be made at a public auction; (c) The creditor notifies the debtor and the owner of the thing pledged of the public auction stating the amount for which the public sale is made; (d) If at the first auction the thing is not sold, a second one with the same formalities shall be held; (e) If at the second auction there is still no sale, the creditor may appropriate the thing pledged and give an acquittance of his entire claim. (Art. 2112, CC) 81. What is the basis of payment of an obligation in case of extraordinary inflation? Extraordinary inflation exists when there is a decrease or increase in the purchasing power of the Philippine currency, and such increase or decrease could not have been reasonable foreseen or was manifestly beyond the contemplation of the parties at the time of the establishment of the obligation. 82. What are the rules for the liquidation of the absolute community of property or conjugal partnership of gains in case of death of a spouse? The rules regarding the liquidation of the absolute community or conjugal partnership are the same. They are as follows:

1. If a special proceeding for the settlement of estate of deceased persons under the Rules of Court has been instituted after the death of one spouse, the absolute community or conjugal partnership shall be liquidated in the said proceeding. 2. If no special proceeding for the settlement of estate of the deceased spouse is instituted, the surviving spouse shall liquidate the absolute community or conjugal partnership either judiciary or extra-judiciary within one year from the death of the spouse. 3. If no liquidation is made within one year from the death of the deceased spouse, any disposition or encumbrance involving any community or conjugal property of the terminated marriage shall be void. 4. Should the surviving spouse contract a subsequent marriage without liquidating the community property or conjugal partnership, a mandatory regime of complete separation of property shall govern the property relations of the subsequent marriage. This is to protect the heirs of the deceased spouse. (Art. 103 and 130 FC) 83. When are extraordinary applicable?

the

effects of inflation

The effects of extraordinary inflation are applicable only where

I press toward BAR OPERATIONS 2013 the mark for the

prize of the high calling of God in Christ Jesus. - Philippians 3:14

BARRISTERS’ CLUB

Page 29 of 67

in CIVIL LAW Prepared by: Atty. Roney Jone P. Gandeza there is an official declaration to that effect by competent authorities. QUESTIONS 84-85 based on the following situation:

are fact

Mayor Alfonso Favis of Magsingal, Ilocos Sur was on board a Toyota Fortuner when the vehicle, cruising at high speed, accidentally hit the minor, Marvin, along the national highway in Vigan City. Despite medical attention, Marvin died six days after the accident. The vehicle was then driven by Rodel, an employee of the municipal government of Magsingal, and registered in the name of Virgilio, a friend of Mayor Favis. 84. Is Mayor Favis solidarily liable for the negligence of the driver? Mayor Favis is not liable. The doctrine of vicarious liability or imputed liability finds no application in the present case. Mayor Favis was neither the employer of Rodel nor the vehicle’s registered owner. There existed no causal relationship between him and Rodel or the vehicle used that will make him accountable for Marvin’s death. Mayor Favis was a mere passenger at the time of the accident. Parenthetically, it has been held that the failure of a passenger to assist the driver by providing him warnings or by serving as lookout does not make the passenger liable for the latter’s negligent acts. The driver’s duty is not that may be delegated to others.

85. Who is liable for Marvin’s death? Liability attaches to the registered owner (Virgilio), the negligent driver (Rodel) and the latter’s employer (Municipality of Magsingal). Settled is the rule that the registered owner of a vehicle is jointly and severally liable with the driver for damages incurred by passengers and third persons as a consequence of injuries or death sustained in the operation of said vehicles. Regardless of whom the actual owner of the vehicle is, the operator of record continues to be the operator of the vehicle as regards the public and third persons, and as such is directly and primarily responsible for the consequences incident to its operation. (Jayme v. Apostol, G.R. No. 163609, November 27, 2008) 86. For damage or injuries arising out of negligence in the operation of a motor vehicle, what is the nature of the liability of the registered owner? For damage or injuries arising out of negligence in the operation of a motor vehicle, the registered owner may be held civilly liable with the negligent driver either: 1) subsidiarily, if the aggrieved party seeks relief based on a delict or crime under Articles 100 and 103 of the Revised Penal Code; or 2) solidarily, if the complainant seeks relief based on a quasidelict under Articles 2176 and 2180 of the Civil Code.

I press toward BAR OPERATIONS 2013 the mark for the

prize of the high calling of God in Christ Jesus. - Philippians 3:14

BARRISTERS’ CLUB

Page 30 of 67

in CIVIL LAW Prepared by: Atty. Roney Jone P. Gandeza It is the option of the plaintiff whether to waive completely the filing of the civil action, or institute it with the criminal action, or file it separately or independently of a criminal action; his only limitation is that he cannot recover damages twice for the same act or omission of the defendant. (PCI Leasing and Finance, Inc v. UCPB General Insurance Co. Inc., G.R. No. 162267, July 4, 2008)

square meter portion of his lot sis occupied by B’s house.

87. D owes C P100, 000. Upon maturity of the loan, D fails to pay and so C sues him in a complaint for sum of money. D answers the complaint and before actual hearing, C assigns the promissory note signed by D to E for P80, 000. The assignee, E, now demands payment from D. For how much is D obliged to pay E?

No, he is not. Article 448 of the Civil Code is unequivocal that the option to sell the land on which another builds, plants or sows in good faith, belongs to the landowner. This advantage in Article 448 of the Civil Code is accorded the landowner because “his right is older, and by the principle of accession, he is entitled to the ownership of the accessory thing.” (Benitez v. C, 226 SCRA 242)

D is liable only for P80, 000, plus cost and interest. Under Article 1634 of the Civil Code, when a credit in litigation is sold, the debtor shall have a right to extinguish it by reimbursing the assignee for the price the latter paid therefor, the judicial costs incurred by him, and the interest on the price from the day on which the same was paid. In paying only such amount, D exercises his right of legal redemption.

89. A owns a parcel of registered land adjacent to that of B. A builds a house on his lot. Unknown to A, however, a portion of his house has encroached on B’s property. After the construction, A sold his lot to C.

(RJPG: The right of redemption granted in Article 1634 of the Civil Code is proper only in case of sale [of the credit in litigation], and not to cases of barter, donation, or other modes of acquisition.) 88. A purchased a property adjacent to that of B. Shortly thereafter, A discovered after relocation survey that a 30-

Despite repeated demands, B refused to vacate the encroached portion. He claims that under Article 448 of the Civil Code, he has the pre-emptive right to purchase the portion encroached upon. Is B correct?

Is A a builder in good faith under Article 448 of the Civil Code? Yes, A can be considered a builder in good faith even if his lot is covered by a Torrens Title. Good faith is a state of mind. Unless one is versed in the science of surveying, no one can determine the precise extent or location of his property by merely examining his paper title. 90. Is C, A’s buyer, entitled to avail of the benefits under Article 448?

I press toward BAR OPERATIONS 2013 the mark for the

prize of the high calling of God in Christ Jesus. - Philippians 3:14

BARRISTERS’ CLUB

Page 31 of 67

in CIVIL LAW Prepared by: Atty. Roney Jone P. Gandeza Yes. Upon delivery of the property to him, C acquired ownership of the property and he is deemed to have stepped into the shoes of the seller, A, in regard to all the rights of ownership, including the right to compel B, the owner of the lot encroached upon, to exercise either of the two options granted to him by law after payment of proper indemnity or to sell the portion encroached upon. 91. Is B entitled to demand the removal of the encroaching structure? B, the lot owner, cannot demand the removal of the encroaching structure. Such right is available only if and he chooses to compel the builder to buy the land at a reasonable price but the latter fails to pay it. (Technogas Phil. V. Court of Appeals, 268 SCRA 5 [1997]) (RJPG: If B decide to appropriate the improvements, the builder has the right to retain the lot until he is paid of his necessary and useful expenses. He is not even required to pay rentals in the meantime.) 92. Aragon is indebted to Benitez and Chua in the amount of P200, 000. Upon maturity of the debt, Aragon fails to pay and so Benitez and Chua sue him in a complaint for sum of money. Aragon answers the complaint and before actual hearing, Benitez assigns his right to the credit to Chua (presumably ½ or P100, 000) for only P75, 000. For how much is Aragon obliged to pay Chua?

Aragon is liable to pay P200, 000 to Chua because the assignment was made to a co-owner. In other words, Aragon cannot redeem the credit in litigation sold by Benitez to Chua. Article 1635 of the Civil Code enumerates the three instances when the debtor cannot redeem a credit in litigation which is sold by his creditor, one of them being an assignment or sale to a co-owner. 93. XYZ Insurance Company insured Pedro’s house for Php 500, 000. In the policy, the insurer undertakes, upon total loss, to either pay the insure value of the house, or rebuild it, upon proof of total loss. If during the life of the policy the insure property is completely destroyed, may the insured insist the insurance company rebuild his house than being paid its insured value? No, because in alternative obligations, the right of choice is given to the debtor, unless it has been expressly granted to the creditor (par. 1 Art 1200, Civil Code). In the absence of an agreement in the insurance policy giving the right of choice to the insured, the general rule applies, and therefore, the insurer may choose which of the two prestations to perform, the performance of one being sufficient. 94. In 1994, Sonia, a Filipina, went to work as a nurse in New York. There she met and fell in love with Peter, an American, whom she married in 1996. Sonia acquired American citizenship in 1998. Due to irreconcilable differences, however, the couple parted ways in 2000. The following year, Peter obtained a

I press toward BAR OPERATIONS 2013 the mark for the

prize of the high calling of God in Christ Jesus. - Philippians 3:14

BARRISTERS’ CLUB

Page 32 of 67

in CIVIL LAW Prepared by: Atty. Roney Jone P. Gandeza valid divorce decree in Nevada. Heartbroken, Sonia repatriated to the Philippines in 2001 and reacquired Filipino citizenship that same year. Sonia subsequently filed an action against Peter for declaration of nullity of marriage under Article 36 of the Family Code with prayer for support pendent lite. Peter at that time has settled in the Philippines with his new wife Maria, a Filipina.

widow remarried. Is she entitled to the inheritance?

Will the case prosper?

96. H instituted his widowed sister as his heir on condition that the latter will not marry again. When H died, his sister remarried. Is she entitled to the inheritance?

The case will not prosper. In Bayot v. Court of Appeals, G.R. No. 155635, November 7, 2008, the Supreme Court said that the divorce decree secured by Peter in Nevada shall be given a res judicata effect in this jurisdiction. As an obvious result of the divorce decree obtained, the marital vinculum between Peter and Sonia is considered severed; they are both free from the bond of matrimony. In plain language, Peter and Sonia are no longer husband and wife to each other. Consequent to the dissolution of the marriage, Peter could no longer be subject to a husband’s obligation under the Family Code. He cannot, for instance, be obliged to live with, observe respect and fidelity, and render support to Sonia. With the valid foreign divorce secured by Peter, there is no marital tie binding him to Sonia. There is on fine no more marriage to be dissolved or nullified. 95. H designated his wife as his universal heir (no other compulsory heirs existed) on condition that when she became a widow, she must never remarry. Two years after H died, the

The condition is valid insofar as the free portion is concerned, since this absolute prohibition was imposed by a deceased spouse – but is not valid insofar as her legitimate is concerned. Therefore, her remarriage makes her lose the free portion, but not the legitimate (for ordinarily, no condition can be imposed on the legitime. (Art. 874, CC)

Yes, because the condition is deemed not written. The rule under Article 874 of the Civil Code is that an absolute condition not to contract a first or subsequent marriage shall be considered as not written unless such condition has been imposed on the widow or widower by the deceased spouse, or by the latter’s ascendants or descendants. The condition that the sister shall not remarry for her to get the inheritance is void because it is contrary to good morality and public policy. QUESTIONS 97-98 based on the following situation:

Jessica institutes an action against Zandro for support in the allegation that they are husband and wife and that Zandro “has reneged on his obligation to financially sipport her as his wife and Tricia as their child.” Zandro denies that he is married to Jessica. He avers that their

I press toward BAR OPERATIONS 2013 the mark for the

prize of the high calling of God in Christ Jesus. - Philippians 3:14

BARRISTERS’ CLUB

are fact

Page 33 of 67

in CIVIL LAW Prepared by: Atty. Roney Jone P. Gandeza marriage is void because it was solemnized without a marriage license; that the marriage ceremony was facilitated by an affidavit wherein they falsely stated that they had been living together as husband and wife for at least five years; and that they have never cohabited as husband and wife. 97. May a party to an action collaterally attack the validity of a marriage as what Zandro had asserted in his answer? Yes. The court has jurisdiction to determine the validity of the marriage of Zandro and Jessica. More appropriately, the validity of their void marriage may be collaterally attacked. Thus, in Nicdao Carino v. Yee Carino, 403 SCRA Phil. 861 [2001], the Supreme Court declared that courts are clothed with sufficient authority to pass upon the validity of two marriages despite the main case being a claim for death benefits. Reiterating Ninal v. Badayog, 384 Phil. 661 [2000], the Supreme Court held that the courts may pass upon the validity of marriage even in a suit not directly instituted to question the validity of said marriage, so long as it is essential to the determination of the case. However, evidence must be adduced, testimonial or documentary, to prove the existence of grounds rendering such a marriage an absolute nullity. In the instant case, it is clear that Zandro and Jessica did not have a marriage license when they contracted their marriage. Instead, they presented a false affidavit stating that they had been living together for more than five years. For

this reason, their marriage is deemed void ab initio. 98. What is the legal standing of the false affidavit executed by Zandro and Jessica? The falsity of the affidavit cannot be considered as a mere irregularity in a formal requisite of marriage. The law dispenses with the marriage license requirement for a man and woman who have lived together and exclusively with each other as husband and wife for a continuous and unbroken period of at least five years before the marriage. The false affidavit which Zandro and Jessica executed so they could push through with the marriage has no value whatsoever; it is a mere scrap of paper. They were not exempt from the marriage license requirement. Their failure to obtain a present marriage license renders their marriage void ab initio. (De Castro v. De Castro, GR No. 160172, February 13, 2008) 99. X obtains a loan from Y. They agree that upon maturity of the loan, X will give either the sum owed or a particular house and lot. X now assails the agreement as a pactum commissorium. Is X correct? The stipulation is simply an alternative obligation, which is expressly allowed by the law. The agreement to convey the house and lot in the event of X’s failure to pay the debt in money at its maturity does not constitute pactum commissorium. It is NOT an attempt to permit the creditor, Y, to declare forfeiture of ther security upon the failure of the

I press toward BAR OPERATIONS 2013 the mark for the

prize of the high calling of God in Christ Jesus. - Philippians 3:14

BARRISTERS’ CLUB

Page 34 of 67

in CIVIL LAW Prepared by: Atty. Roney Jone P. Gandeza debtor to pay the debt at maturity. It simply provided that if the debt is not paid in money, it shall be paid another way. 100. D borrowed from C P5, 000 payable in one year. When C was abroad, C’s 16-year old son borrowed P2, 500 from D for his school tuition. However, the son spent the money on a cellular phone. When the debt to C fell due, D tendered only P2, 500 claiming compensation on the P2, 500 borrowed by C’s son. Is D legally entitled to claim partial legal compensation? No. This is so because under Articles 1278 and 1279 of the Civil Code, in order that there will be a valid and effective compensation, it is essential that there must be two parties who in their own right are principal creditors and principal debtors of each other. In the instant case, C cannot be considered as a party to the act of his son in borrowing P2, 500 from D. Consequently, he did not become a principal debtor of D; neither did D become a principal creditor of C. Therefore, there can be no partial compensation of the P5, 000 borrowed by D from C. 101. Would the answer be the same if C’s son actually used the money for his school tuition? There would be no difference in the answer. The fact that C’s son actually used the P2, 500 for his school tuition did not make C a party to the contract between his son and D. Therefore, C is not the principal debtor of D and D is not the principal

creditor of C with respect to the subject amount. 102. A and B are neighbors. On his building’s wall, A opened a window beneath the ceiling joists to admit light in 1995. Even after 10 years, B may still obstruct the light by constructing on his own lot a building higher than A’s unless A makes a notarial prohibition prohibiting B from making the construction. Of in 23001 A makes the prohibition, may still B make the obstruction in 2007? Yes, because it is only in 2013 (ten years after the notarial prohibition) when A may be said to have acquired the negative easement of light and view. After 2013, B may no longer obstruct. 103. About 15 years ago, A constructed a house on his lot adjoining the lot owned by B. He provided it with windows overlooking B’s lot half a meter away from the boundary line. A month ago, B brought an action against A for the closure of the windows on the allegation that they violate the law on distances. Has A acquired an easement of light of view by prescription of ten years? No. In the first place, there was no formal prohibition under which A prohibited B from obstructing his light and view. In the second place, A did not observe the legal requirement that there should be a distance of at least two meters between the windows and B’s lot, since the view is direct.

I press toward BAR OPERATIONS 2013 the mark for the

prize of the high calling of God in Christ Jesus. - Philippians 3:14

BARRISTERS’ CLUB

Page 35 of 67

in CIVIL LAW Prepared by: Atty. Roney Jone P. Gandeza According to the Civil Code, nonobservance of this distance does not give rise to prescription. 104. If the vendee in a contract of sale expressly renounces the right to warranty in case of eviction, and his eviction should take place, can he still hold the vendor liable? It depends. In case or renunciation of the warranty without knowledge of the risks of eviction, the vendor is only bound to pay the value of the thing at the time of the eviction. Although as a consequence of the waiver, the vendor is not bound to indemnify the vendee fully in accordance with Article 1555 of the Civil Code still the effect of the waiver cannot be extended as to exempt the vendor from returning the price. When eviction occurs, the contract is left without cause as to the vendee and inasmuch as his obligation to pay the price is condition upon the delivery of the thing by the vendor, from the moment the vendee is deprived of the possession of the thing, the payment of the price really becomes a payment of what is not due which, under Article 2154 of the Civil Code, should be returned. 105. A appoints B to sell his registered land. B negotiates the sale with C. Unknown to B, A also negotiates the sale with D, thereby making contracts incompatible with each other. Who is now the owner of the land, C or D? Article 1916 of the Civil Code establishes the rule of preference when two persons contract with regard to the same thing, one of them with the agent and the other with the

principal. Under this provision, that of prior date shall be preferred, without prejudice to the provisions of Article 1544 of the Civil Code. Consequently, whoever between C and D registers the sale on good faith is the owner of the land; and in the absence of registration, the first to take possession of the land in good faith. 106. While sitting on his front porch, Abe watched three men paint a fence on his property. The men later learned that they had made a mistake. The fence they were to have painted was in the next block, in the property of Rey. Is Abe obliged to pay the painters? By his toleration of the trespassers and by his failure to protest, Abe is obliged to pay the painters the reasonable value of the work done and the materials used. The basis of his liability would be his implied acceptance of the offer of the painters to paint his fence. In other words, the basis of Abe’s liability is his implied contract with the painters. (RJPG: It is error to state that Abe is liable because of the benefit bestowed; or that his liability is grounded on the existence of a quasicontract; or that he is obliged to pay the painters to prevent unjust enrichment. This is so because of the implied contract between him [Abe] and the painters.) 107. Suppose Abe had not been at home, and he did not recover that the fence had been painted until he returned in the evening, is Abe liable? If so, what would be the basis of his liability?

I press toward BAR OPERATIONS 2013 the mark for the

prize of the high calling of God in Christ Jesus. - Philippians 3:14

BARRISTERS’ CLUB

Page 36 of 67

in CIVIL LAW Prepared by: Atty. Roney Jone P. Gandeza Abe would not be obliged in this situation. HE did not learn of the benefits bestowed until after they had been completed. There had been no communication of the offer of the painters and no express or implied acceptance by Abe of the work done. Neither is Abe liable to the painters under quasi-contract. This is so because there was neither a case of colution indebiti or negotiorum gestio in the instant case. 108. Apolinario Mabini died intestate in 1995, leaving his wife, Dorothy, four legitimate children, and considerable properties which they divided among themselves. Claiming to be an illegitimate son of the deceases Apolinario, and having been left out in the extrajudicial settlement of Apolinario’s estate, Mario instituted an action for partition against Dorothy and her children. At the trial, Mario admitted that he had none of the authentic documents mentioned in Article 172 of the Family Code to show that he was the illegitimate child of Apolinario. Is this admission sufficient basis for Dorothy and her children to move for the dismissal of Mario’s complaint? Yes. An illegitimate child, like Mario, is allowed by law to establish his illegitimate filiation either by an authentic document or by “any other means allowed by the Rules of Court and special laws,” like his baptismal certificate, a judicial admission, a family Bible in which his name has been entered, common reputation respecting his pedigree, admission by silence, testimonies of witnesses, and

other kinds of proof admissible under Rule 130 of the Rules of Court. The problem of Mario, however, is that, since he seeks to prove his filiation under the second paragraph of Article 172 of the Family Code, his action is now barred because of his alleged father’s death in 1995. This particular provision of law specifically provides that when the action to claim illegitimate filiation is based on the second paragraph of Article 172, the action must be brought during the lifetime of the alleged parent. It is clear, therefore, that Mario can no longer be allowed at this time to introduce evidence of his open and continuous possession of the status of an illegitimate child or prove his alleged filiation through any of the means allowed by the Rules of Court or special laws. The simple reason is that Apolinario is already dead and can no longer be heard on the claim of his allged son’s illegitimate filiation. (Uvguanco v. Court of Appeals. G.R. No. 76873, Ocotber 26, 1989) 109. Rey is a registered guest at the Pines View Hotel in Baguio City. In the middle of the night, Abe, a hotel guest himself, went up the fire escape, slowly raised the window in Rey’s room, went inside the room, and stole Rey’s expensive laptop. The hotel disclaimed liability on the ground of force majeure. Is the hotel liable? Yes. The problem involves a case of robbery with force upon things. Under Article 2001 of the Civil Code, the act of a thief or robber, who has entered the hotel is not deemed force majeure, unless it is done with

I press toward BAR OPERATIONS 2013 the mark for the

prize of the high calling of God in Christ Jesus. - Philippians 3:14

BARRISTERS’ CLUB

Page 37 of 67

in CIVIL LAW Prepared by: Atty. Roney Jone P. Gandeza the use of arms irresistible force.

or

through

an

110. Abe is a bell boy at the Pines View Hotel. At the point of a gun, he entered Rey’s hotel room and robbed Rey and his wife of their valuables. The hotel disclaimed liability on the ground of force majeure. Is the hotel liable? Yes. It is true that here the robbery was committed with the use of arms, (which would have been a case of force majeure) but then Abe was an employee of the hotel. The hotel will be liable not because of force majeure under Article 2001 which evidently refers to a stranger, but because of Article 2000 of the Civil Code which provides that the master is responsible for the acts of his servants. 111. Arnel pledges his Nokia cellular phone to Bert to secure a loan. Arnel and Bert agree that the latter could purchase the cellular phone at the current purchase price if the debt is not paid on time. Arnel assails the agreement for being a pactum commissorium. Is Arnel correct? No. The stipulation is not a pactum commissorium. What is prohibited by Article 2008 of the Civil Code, dealing with pactum commissorium, is the automatic appropriation by the creditor or pledge of the thing pledged in payment of the loan at the expiration of the period agreed upon. Where there is an express authorization on the creditor to purchase the thing pledged at the current market price, the contract would not come within the prohibition as there is no

automatic appropriation by the creditor of pledge of the thing pledged. 112. Is there an instance under the law where the creditor is allowed to appropriate for himself the thing given by way of security because of non-payment of the debt? Yes, in the case of pledge. If at the first auction the thing pledged is not sold, a second auction must be held. If the thing pledged is not sold at second auction, the creditor is now allowed to appropriate for himself the thing pledged but he must give an acquittance for his entire claim. (Art. 2112, Civil Code) 113. When the proceeds of the sale of the mortgaged property in chattel mortgage does not fully satisfy the debt, is the mortgage entitled to recover the deficiency from the mortgagor? Yes. It is a settled rule that if the proceeds of the sale are insufficient to cover debt either in an extrajudicial or judicial foreclosure of mortgage, the mortgagee is entitled to claim deficiency from the debtor. While the legislature has denied the right of a creditor to sue for deficiency resulting from foreclosure of security given to guarantee an obligation as on the case of pledges (Art. 2115, Civil Code) and in chattel mortgages of a thing sold on installment basis (Art. 1484, par. 3, Civil Code), and the law does not prohibit recovery of deficiency. Accordingly, a deficiency claim arising from the extrajudicial foreclosure of mortgage is allowed. (PNB v. Court of Appeals, 308 SCRA 229 [1999])

I press toward BAR OPERATIONS 2013 the mark for the

prize of the high calling of God in Christ Jesus. - Philippians 3:14

BARRISTERS’ CLUB

Page 38 of 67

in CIVIL LAW Prepared by: Atty. Roney Jone P. Gandeza 114. May a legitimate child impugn his legitimate status? No, the law itself establishes the legitimacy of a child conceived or born during the marriage of his parents. The presumption of legitimacy fixes a civil status for the child born in wedlock, and only the father (Art. 160, Family Code), or in exceptional instances the latter’s heirs (Art. 171, Family Code), can contest in an appropriate action the legitimacy of a child. A child cannot choose his own filiation. QUESTIONS 115-116 based on the following situation:

are fact

Rey and Mimi were married in 1985. They were childless. By stroke of fate, an infant whose parents were unknown was entrusted to them by a friend. Eager to have a child of their own, Rey and Mimi registered the child to make it appear that they were the child’s parents. They named the infant Angela and followed Rey’s surname. The spouses reared and cared for Angela as if she was their own. They sent the child to exclusive schools, and used the surname of Rey on all her school records and documents. Rey died in 1998. In 2000, Mimi married Abe, an American citizen. Shortly thereafter, Mimi decided to adopt Angela by availing of the amnesty given under the law to those individuals who simulated the birth of a child. Thus, in 2002, Mimi filed a petition for the adoption of Angela. Abe executed an affidavit giving his consent to the adoption of Angela.

115. Can Angela?

Mimi

alone

Mimi alone cannot adopt Angela. At the time the petition for adoption was filed, Mimi had already remarried. She filed the petition by herself, without being joined by her husband, Abe. The law is explicit. Section 7, Article III of RA No. 8552 specifically provides that the husband and wife shall jointly adopt, except if one spouse seeks to adopt the legitimate child of the other; or if one spouse seeks to adopt his own illegitimate child; or if the spouses are legally separated from each other. Mimi does not fall under any of the three exceptions enumerated in Section 7. First, the child adopted is not the legitimate child of Mimi or of her husband Abe. Second, the child is not the illegitimate child of Mimi. And third, Mimi and Abe are not legally separated from each other. 116. Suppose Abe filed for divorce during the adoption proceedings, would that make a difference in you answer? It will not make any difference in my answer. The filing of a case for dissolution of the marriage between Mimi and Abe is of no moment. It is not equivalent to a decree of dissolution of marriage. Until and unless there is a judicial decree for the dissolution of the marriage between Mimi and Abe, the marriage still subsists. That being the case, joint adoption by the husband and the wife is required. To repeat; since at the time the petition for adoption was filed, Mimi was married to Abe, joint adoption is mandatory. (In re: Petition

I press toward BAR OPERATIONS 2013 the mark for the

prize of the high calling of God in Christ Jesus. - Philippians 3:14

BARRISTERS’ CLUB

adopt

Page 39 of 67

in CIVIL LAW Prepared by: Atty. Roney Jone P. Gandeza for adoption of Michele P. Lim GR Nos. 168992-93 May 21, 2009) 117. Venus secures a judgment against Erlinda for the payment of civil liability arising from the crime of slander committed by Erlinda against Venus. To satisfy the judgment, the sheriff levies on a real property owned by the conjugal partnership of Erlinda and her husband, Romulo. May the property be sold on execution? The property is exempt from execution. Article 122 of the Family Code explicitly provides that payment of personal debts contracted by the husband or the wife before or during the marriage shall be charged to the conjugal partnership, except insofar as they redounded to the benefit of the family. By no stretch of imagination can it be concluded that the civil obligation arising from the crime of slander committed by Erlinda redounded to the benefit of the conjugal partnership. Unlike in the system of absolute community where liabilities incurred by either spouse by reason of a crime or quasi-delict is chargeable to the absolute community of the property, in the absence or insufficiency of the exclusive property of the debtorspouse, the same advantage is not accorded in the system of conjugal partnership of gains. The conjugal partnership of gains has no duty to make advance payments for the liability of the debtor-spouse. (Sps. Buado v. Court of Appeals, 145222, April 24, 2009)

118. What is the doctrine of dependent relative revocation in the law on succession? If the testator revokes his will with the present intention of making a new one and the new will is not made, or if made, fails to effect for any reason whatsoever, it will be presumed that the testator prefers the old will to intestacy. The old will can still be admitted to prbate. Stated otherwise, the revocation is subject to a SUSPENSIVE CONDITION: That the testator will make a new will and that such will shall take effect. If such condition is not fulfilled, then there is no revocation. 119. The Domestic Adoption Act (RA 8552) requires, inter alia, that before an alien can adopt in the Philippines, he must have resided in the Philippines for at least three continuous years prior to the filing of the application for adoption and maintains such residence until the adoption decree is entered. Are there exceptions to this requirement? Yes, they are as follows: (a) The alien is a former Filipino citizen who seeks to adopt a relative within the fourth civil degree of consanguinity; (b) The alien seeks to adopt the legitimate or illegitimate child of his or her Filipino spouse; and (c) The alien is married to a Filipino citizen and seeks to adopt jointly with his or her spouse a relative within the fourth civil degree

I press toward BAR OPERATIONS 2013 the mark for the

prize of the high calling of God in Christ Jesus. - Philippians 3:14

BARRISTERS’ CLUB

Page 40 of 67

in CIVIL LAW Prepared by: Atty. Roney Jone P. Gandeza of consanguinity or affinity of the Filipino spouse. 120. Abe is the son of Fidel with his first wife, while Rey is Fidel’s son with his second wife. Both wives predeceased Fidel. Upon Fidel’s death in 2008, Abe immediately instituted and action for partition of Fidel’s estate. After trial in due course, the court rendered judgment ordering the partition in equal shares between Abe and Rey the land exclusively owned by Fidel. As Abe and Rey failed to agree on how to partition the property, the court ordered its sale at public auction. However, the public sale did not push through because Rey refused to include in the auction sale the house standing on the land on the allegation that the house has been residence nearly 20 years, and has thus acquired the status of a family home. Rey also pointed out that since the house was not mentioned in Abe’s complaint for judicial partition, such house is not susceptible of partition. Decide. The house is deemed included in the judgment of partition, and this is true even if its existence was not mentioned in Abe’s complaint. Pursuant to law, since Fidel owned the land, he also owned the house which is a mere accessory to the land. Both properties form part of the estate of the deceased and are held in co-ownership by his heirs. Any decision in the action for partition of said estate would cover not just the subject land but also the subject house.

While it is conceded that the subject house is covered by the judgment of partition, the ruling does not necessarily countenance the immediate and actual partition of said property by way of public auction in the view of the suspensive prescription imposed under Article 159 of the Family Code. Set against the foregoing rule, the family home – consisting of the subject house and lot on which it stands- cannot be partitioned at this time, even if it has passed to the coownership of his heirs. Fidel died in 2008. Thus, for 10 years from said date or until 2018, or for a longer period, if there is still a minor beneficiary residing therein, the family home he constituted cannot be partitioned. (Arriola et. al. v Arriola, GR No. 177703, January 28, 2008) (RJPG: The Supreme Court declared in Honrado v. Court of Appeals, 476 SCRA 280 [2005], that a claim for exception from execution or forced sale under Article 153 should be set up and proved before the sale of the property at public auction. In the above-entitled case of Arriola, Dos timely objected to the inclusion of the subject house although for a different reason. 121. Distinguish between right of redemption and equity of redemption. Equity of redemption is the right of the mortgagor after judgment in a judicial foreclosure to redeem the property by paying to the court the amount of the judgment debt before the sale or confirmation of the sale. On the other hand, right of redemption is the right of the mortgagor to redeem the property

I press toward BAR OPERATIONS 2013 the mark for the

prize of the high calling of God in Christ Jesus. - Philippians 3:14

BARRISTERS’ CLUB

Page 41 of 67

in CIVIL LAW Prepared by: Atty. Roney Jone P. Gandeza sold at an extrajudicial foreclosure sale by paying to the buyer in the foreclosure sale the amount paid by the buyer within one year from such sale. 122. May the true owner of a movable property recover possession of his property from the present possessor? If so, is there a need to reimburse said possessor? The true owner of a movable property may recover possession of his property without reimbursement from a possessor in bad faith or even from possessor in good faith if said owner had lost the property or been unlawfully deprived of it, the acquisition being from a private person. The owner may also recover possession of his movable property but should reimburse the possessor if such possessor acquired the property in good faith at a public sale or auction (Art. 559, CC) However, the owner can no longer recover possession of his movable property, even if he offers to reimburse, whether or not he had lost his property or had been unlawfully deprived of it, if the possessor had acquired the property in good faith by purchase from a merchant’s store, or in fairs, or markets in accordance with the Code of Commerce and Special Laws (Art. 1505, Civil Code and Arts. 85, 86 Code of Commerce); or if the owner is precluded from denying the seller’s authority, or if the possessor had obtained the goods because he was an innocent purchaser for value and a holder of a negotiable document title to the goods. (Art. 1518, CC)

123. Why is a negative easement acquirable by prescription despite its non-apparent? Generally, negative easements cannot be acquired by prescription because they are non-apparent. Still, the very existence of the Civil Code (insofar as it relates to negative easements), prove that in certain cases, and for purposes of prescription, there are negative easements that may be indeed be considered “apparent”, not because there are visible signs of existence but because of the making a notarial prohibition. The notarial prohibition makes apparent what really is nonapparent. 124. While walking along Session Road in Baguio City, A found a purse containing P50, 000. Can A become the owner of the P50, 000? If A knows the owner of the purse, there is no way by which he can become the owner of the P50, 000 because under the law, he must return the purse, including its contents, to the owner. If the owner is unknown, A shall immediately deposit the movable with the mayor of the place where the purse was found. There shall then be a public announcement of the finding for two consecutive weeks. Six months from the publication having elapsed without the owner having appeared, the thing found shall be awarded to A, after reimbursement of the expenses. (Art. 719, CC) Should the owner appear in time, he shall be obliged to pay A, as a reward, 1/10 of the amount found. (Art. 720, CC)

I press toward BAR OPERATIONS 2013 the mark for the

prize of the high calling of God in Christ Jesus. - Philippians 3:14

BARRISTERS’ CLUB

Page 42 of 67

in CIVIL LAW Prepared by: Atty. Roney Jone P. Gandeza 125. When can the owner of an estate claim a compulsory right of way? Under Articles 649 and 650 of the Civil Code, the owner of an estate may claim a compulsory easement of right of way after he has established the existence of the following requisites: (a) the estate is surrounded by other immovables and is without an adequate outlet to a public highway; (b) proper indemnity is paid; (c) the isolation is not due to the proprietor’s own acts; and (d) the right of way claimed is at a point least prejudicial to the servient estate and insofar as consistent with the law, where the distance from the dominant estate to a public highway may be the shortest. 126. May an existing easement of right of way be extinguished by the opening of an adequate outlet to a public highway? An easement of right of way provided for in a contract of sale is a voluntary easement. As such, it cannot be extinguished by the opening of an adequate outlet to a public highway. The opening of an adequate outlet can extinguish a legal or compulsory easement but not a voluntary easement. (La Vista Association v. Court of Appeals, 278 SCRA 498 [2000]) 127. B donated to C a parcel of land subject to a condition. When C failed to comply with the condition, B sold the land to D. Is the sale an act of revocation of the donation? No. The act of selling the property donated cannot be considered as a valid act of

revocation of the donation because a formal action in court to revoke the donation must be filed by the donor pursuant to Article 764 of the Civil Code which speaks of an action that has a prescriptive period of four (4) years from non-compliance with the condition stated in the donation. The rule that there can be automatic revocation without the benefit of a court action does not apply in this case because the subject donation is devoid of any provision providing for automatic revocation in the event of con-compliance with the condition set forth therein. Thus, a court action is necessary to be filed within four (4) years from the non-compliance with the condition violated. 128. May the heirs of the donor sue the heirs of the done for revocation of the donation if there is a violation of any restriction in the deed of donation? Yes. Under Article 764 of the Civil Code, the donor or his heirs have the personality to question the violation of any restriction in the deed of donation. Consequently, the right to revoke be transmitted to the heirs of the donor and may be exercised against the heirs if the done, and the action prescribes after four years from the violation of the condition. 129. A lease agreement contains the following stipulation: “That the term of the lease shall be 14 years beginning April 1, 1998 and may be renewed for a like term at the option of the lessee.” The lessor contends that this stipulation is void because it is violative if the principle of mutuality of contracts under

I press toward BAR OPERATIONS 2013 the mark for the

prize of the high calling of God in Christ Jesus. - Philippians 3:14

BARRISTERS’ CLUB

Page 43 of 67

in CIVIL LAW Prepared by: Atty. Roney Jone P. Gandeza Article 1308 of the Civil Code. Is the lessor correct? No. The fact that the lessee’s option to renew the lease is binding only on the lessor and can be exercised only by the lessee does not render such option void for lack of mutuality. After all, the lessor is free to give or not to give the option to the lessee. And while the lessee has a right to elect whether to continue with the lease or not, once he exercises his option to continue and the lessor accepts, both parties are thereafter bound by new lease agreement. Their rights and obligations become mutually fixed, and the lessee is entitled to retain the possession of the property for the duration of the new lease, and the lessor may hold him liable for the rent thereof. Mutuality obtains in such a contract and equality between the lessor and the lessee since they remain with the same faculties in respect to fulfillment. 130. How shall the clause “may be renewed for a like term at the option of the lessee” be interpreted or applied? The clause means that the exercise by the lessee of his option resulted in the automatic extension of the contract of lease under the same terms and conditions prevailing in the original contract of lease, i.e., for 14 years, the phrase “for a like term” referring to the term of the lease. If the renewed contract were still subject to mutual agreement by the lessor and the lessee, then the option – which is an integral part of the consideration for the contract – would be rendered worthless. For then, the lessor could easily defeat the lessee’s right by simply imposing

unreasonable and onerous conditions to prevent the parties from reaching an agreement. (Allied Banking Corp. v. Court of Appeals, 284 SCRA 357 [1998]) 131. May a lessee invoke the application of Article 16887 of the Civil Code for the extension of his lease in the same ejectment case filed by his lessor? Yes. The exercise of the power given to the courts in Article 1687 of the Civil Code to extend the period of the lease when the lessee has been in occupancy of the premises for more than a year does not contemplate a separate action for that purpose. That power may be exercised as an incident in the action for ejectment itself and by the court having jurisdiction over it. 132. May a common carrier be held liable for the death or injury of a passenger caused by the willful acts or negligence of another passenger or stranger? Yes, a common carrier is responsible for the death or injury of a passenger caused by the willful act or negligence of another passenger or stranger, but only if the employees of the common carrier through the exercise of diligence of a good father of family could have prevented the act or omission. 133. When does an action for reconveyance based on implied or constructive trust prescribe? An action for reconveyance of a parcel of land based on an implied or constructive trust prescribes in ten years, the point of reference of being

I press toward BAR OPERATIONS 2013 the mark for the

prize of the high calling of God in Christ Jesus. - Philippians 3:14

BARRISTERS’ CLUB

Page 44 of 67

in CIVIL LAW Prepared by: Atty. Roney Jone P. Gandeza the date of registration of the deed or the date of issuance of the certificate of title over the property. However, this rule applies only when the plaintiff or the person enforcing the trust is not in possession of the property, since if a person claiming to be the owner thereof is in actual possession of the property, the right to seek reconvenyance which in effect seeks to quiet title to property, does not prescribe. 134. A contract of lease contains a stipulation authorizing the lessor to take over possession of the leased premises without judicial intervention upon failure of the lessee to pay rent. Is the stipulation valid? The validity of a contractuallystipulated termination clause has been upheld by the Supreme Court. The stipulation is in the nature of resolutory condition, for upon the exercise by the lessor of his right to take possession of the leased property, the contract is deemed terminated. This kind of contractual stipulation is not illegal, there being nothing in the law proscribing such kind of agreement. Moreover, judicial permission to cancel the lease agreement is not, therefore, necessary because of the express stipulation in the contract of lease that the lessor, in case of failure of the lessee to comply with the terms and conditions thereof, can take over the possession of the leased premises, thereby cancelling the contract of lease. Resort to judicial action is necessary only in the absence of a special provision granting the power of cancellation.

135. D borrowed from C P350, 000.00 payable in three months. To secure payment of the loan, D executed in C’s favor a Real Estate Mortgage. From the loan amount that D was to receive, P17, 500.00 was pre-deducted as interest for the first month which was equivalent to 5% of the principal debt, and P7, 500.00 equivalent to 1.79% of the debt, was further deducted as service fee. Thus, D actually received only P325, 000. When D failed to pay the loan, the loan was extended through a restructuring agreement in such a way that the unpaid interest became part of the principal. The restructuring agreement adopted all other terms and conditions contained in the first loan agreement. Due to the continued inability of D to pay, the loan agreement was renewed to five more times. In all these subsequent renewals, the same terms and conditions found in the first agreement were retained. Consequently, the principal was finally increased to P880, 000. This increase in the amount of principal had been due to unpaid interest and other charges. When the debt remained unpaid, C formally demanded from D the payment of P880, 000. When the demand was ignored, C foreclosed the mortgage. At the auction sale, the mortgaged property was sold to C, the lone bidder, for the amount of P880, 000. Is the foreclosure sale valid? What is D’s remedy in law?

I press toward BAR OPERATIONS 2013 the mark for the

prize of the high calling of God in Christ Jesus. - Philippians 3:14

BARRISTERS’ CLUB

Page 45 of 67

in CIVIL LAW Prepared by: Atty. Roney Jone P. Gandeza The foreclosure is void. Since D was not given an opportunity to settle his debt, at the correct amount and without the iniquitous interest imposed, no foreclosure proceedings may be instituted. A judgment ordering a foreclosure sale is conditioned upon a finding on the correct amount of the unpaid obligation and the failure of the debtor to pay the said amount. (Secs. 2 and 3, Rule 68, Rules of Court)

for reconveyance. Since the property has not yet been transferred by C to an innocent purchase for value, D may still avail of the remedy of reconveyance. (Espiritu v. landrito, GR Nos. 169617, April 4, 2007) 136. Donor donates on January 1. Donee accepts on January 5. Donor dies on January 8. Acceptance of donation reached donor’s house on January 10. Is there a perfected donation?

In this case, it has not yet been shown that D had already failed to pay the correct amount for the debt, and therefore, a foreclosure sale cannot be conducted in order to answer for the unpaid debt. The foreclosure sale is conducted upon D’s failure to pay P880, 000 is void since the amount demanded as the outstanding loan was overstated, consequently, it has not been shown that D has failed to pay his outstanding obligation.

There is no perfect donation because done died at the time of the “making” of the donation.

As a result, the subsequent registration of the foreclosure sale cannot transfer any rights over the mortgaged property to C. The registration of the foreclosure sale, which is void, cannot vest title over the mortgaged property. The torrens system does not create or vest title where one does not have a rightful claim over a real property. It only confirms and records title already existing and vested. It does not permit one to enrich himself at the expense of the another. Thus, registration or real property by one person in his or her name, whether by mistake or fraud, the real owner being another person, impresses upon the title acquired the character of a constructive trust for the real owner, which would justify an action

(RJPG: The term “making” means “perfection” of the donation; otherwise, if “making” means “giving”, Article 737 of the Civil Code would be inconsistent with Article 734 which states that the donation is perfected from the moment the donor knows of the “acceptance” by the done. Thus, at the time the donation is perfected, both the donor and the done must be capacitated.)

137. Insane donor donates on January 1. Donee accepts on January 5. Donor becomes san on January 7. Donor receives acceptance on January 8. Is there a perfect donation? Yes, because at the time of the “making”, both parties had capacity.

138. A Real Estate Mortgage contains a stipulation that the mortgagee shall send notice of foreclosure proceedings to the mortgagor at the latter’s given address. If the mortgagee decides to extrajudicially foreclose the mortgage, is

I press toward BAR OPERATIONS 2013 the mark for the

prize of the high calling of God in Christ Jesus. - Philippians 3:14

BARRISTERS’ CLUB

Page 46 of 67

in CIVIL LAW Prepared by: Atty. Roney Jone P. Gandeza personal notice to the mortgagor still necessary?

repurchase prescribed.

Yes. In extrajudicial foreclosure proceedings, personal notice to the mortgagor is actually unnecessary unless stipulated. In this case, the mortgagor and the mortgagee voluntarily agreed on an additional stipulation embodied in their mortgage agreement. Not being contrary to law, morals, good customs, public order or public policy, the mortgagee should send notice of the extrajudicial proceedings to the mortgagor in compliance with the stipulation in the mortgage deed. (Tamayo, Jr. v. Heirs of Gavino Dominguez, 498 SCRA 342 [2006])

139. Is B entitled to repurchase the subject property?

QUESTIONS 139-141 are based on the following situation: In 1976, A obtained an original certificate of title covering a parcel of land secured through a homestead patent. Upon A’s death in 1978, the land was transferred by succession to his son B who obtained a transfer certificate of title in his name. In 1989, B mortgaged the land to DBP as security for a loan. When B failed to pay, the bank extrajudicially foreclosed the mortgage, purchased the property at the public auction, and secured a title in its name after consolidation. Invoking Section 119 of the Public Land Act, B tried to repurchase the property in 2002, but the bank refused. The bank defends that there can no longer be any right of repurchase but by a transfer certificate of title, and that in any event, the right to

had

Yes, B is entitled to repurchase. The plain intent of Section 110 of the Public Land Act is to give a homesteader every chance to preserve and keep in the family the land that the State has gratuitously given him as a reward for his labor in cleaning, developing and cultivating it. Hence, the fact that the land had been inherited by the patentee’s son (and a new titled in his name is issued) does not bring it outside the purview of Section 119. In fact, the policy behind the law is fulfilled because the land remains in the family of the patentee. 140. Is B’s right to repurchase already time-barred? The rules on redemption in case of an extrajudicial foreclosure of land acquired under free patent or homestead statutes may be summarized as follows: If the land is mortgaged to parties other than rural banks, the mortgagor may redeem the property within one (1) year from the registration of the certificate of sale pursuant to Act No. 3135. If he fails to do so, he or his heirs may repurchase the property within five (5) years from the expiration of the redemption period also pursuant to Section 119 of the Public Land Act. Consequently, the complaint filed in 2002 was on time. 141. If B is already deceased, may his widow exercise the right of redemption?

I press toward BAR OPERATIONS 2013 the mark for the

prize of the high calling of God in Christ Jesus. - Philippians 3:14

BARRISTERS’ CLUB

already

Page 47 of 67

in CIVIL LAW Prepared by: Atty. Roney Jone P. Gandeza The term “legal heirs” as used in Section 199 is used in generic sense. It is broad enough to cover any person who is called to the succession either by provision of a will or by operation of law. Thus, legal heirs include both testate and intestate heirs depending upon whether succession is by will of the testator or by law. Legal heirs are not necessarily compulsory heirs but they may be so if the law reserves a legitime for them. The interpretation of “legal heirs” is more in keeping with the salutary purpose behind the enactment of Section 119 and the jurisprudence laid down on the matter. Indeed, it is not far-fetched to arrive at a more liberal conclusion if the section is analyzed in accordance with its purpose.

both the donor and the donee must be capacitated.)

The widow inherited the property from B, her husband, who in turn inherited it from his father A. The widow, as daughter-in-law of the patentee, can be considered as among the legal heirs who can repurchase the land. The Supreme Court has time and again said that between two statutory interpretations, that which better serves the purpose of the law should prevail. Furthermore, the law must be liberally construed in order to carry out its purpose. (DBP v. Gagarani, GR No. 172248, September 17, 2008)

143. Article 75 of the Family Code specifically provides that in the absence of a marriage settlement, or when the regime agreed upon is void, the system of absolute community of property shall govern the property relations of the spouses during their marriage. Is this rule absolute?

(RJPG: The term “making” must be interpreted to mean “perfection” of the donation; otherwise, if “making” means “giving”, Article 737 of the Civil Code would be inconsistent with Article 734 which states that the donation is perfected from the moment the donor knows of the “acceptance” by the donee. Thus, at the time the donation is perfected,

142. Y, the prospective husband, gives a donation propter nuptias of a parcel of land to X, his prospective wife. Due to quarrel, the marriage is called off. Is the donation automatically revoked? I distinguish. If the donation propter nuptias is incorporated in X and Y’s settlement, the donation is rendered void by the non-celebration of the marriage (Art. 81, Family Code). However, if the donation is propter nuptias is made independently of the marriage settlement, the donation is revocable only at the instance of the donor. (Art. 86 (1), Family Code)

The rule is not absolute. Pursuant to Article 103 of the family Code, should the surviving spouse contract a subsequent marriage without liquidating the absolute community of property regime or conjugal partnership, a mandatory regime of complete separation of property shall govern the property relations of the subsequent marriage. This is to protect the heirs of the deceased spouse. 144. What unions are covered by Articles 147 and 148 of the Family Code?

I press toward BAR OPERATIONS 2013 the mark for the

prize of the high calling of God in Christ Jesus. - Philippians 3:14

BARRISTERS’ CLUB

Page 48 of 67

in CIVIL LAW Prepared by: Atty. Roney Jone P. Gandeza Article 147 applies to two relationships. The first is when a man and a woman who are capacitated to marry each other live exclusively as husband and wife without the benefit of marriage. The second is when a man and a woman live together under a void marriage where the parties do not have an existing marriage with another. Article 148, on the other hand, applies to five kinds of relationships; namely (1) bigamous marriages; (2) adulterous relationships; (3) relationships in a state of concubinage; and (5) multiple alliances of the same man. 145. X has no child. At the time he gave a donation of P100, 000, he had P1 million. Therefore, after the donation, he had P900, 000 left. Later he adopted a minor child. At the time he made the adoption, he had only P50, 000 left. Should the donation be reduced? How much and within what period? The donation should be reduced by P25, 000 because the legitime is impaired to that extent. The action to revoke or reduce the inofficious donation must be brought within four years from adoption. Thus: 50, 00 (value at the time of adoption) 100,000 (value of donation) 150,000 The adopted child has the same rights as a legitimate child. As such, he is entitled to a legitimate of P75, 000. But because the residue of X’s estate is only P50, 000, the donation is reducible by P25, 000.

146. Explain the doctrine of selfhelp in civil law. The doctrine of self-help in civil law is the right to counter in certain cases, forces with force. It is enunciated under Article 429 of the Civil Code, and it can be exercised by the owner at the time of actual or threatened dispossession. When the possession has already been lost, the owner must resort to judicial process for the recovery of property. 147. Is the rule on accession discrete absolute? The rule is not absolute. The following are the instances when the owner has no right to the fruits of his property: (a) If the thing is in the possession of a possessor in good faith, in which case possessor is entitled to the fruits. (Art. 554, CC) (b) If the thins is subject to a usufruct, in which case the usufructuary is entitled to the fruits. (Art. 556, CC) (c) If the thing is leased, in which case the lessee is entitled to the fruits leased, although such lessee must pay to the owner rentals which are in the nature of civil fruits. (Art. 1654, CC) (d) If the thing is in the possession of an antichretic creditor, in which case such creditor is entitled to the fruits with the obligation of applying them to the interest and principal. (Art. 2132, CC) 148. What are the rules in adjunction? Adjunction is governed by the following rules:

I press toward BAR OPERATIONS 2013 the mark for the

prize of the high calling of God in Christ Jesus. - Philippians 3:14

BARRISTERS’ CLUB

Page 49 of 67

in CIVIL LAW Prepared by: Atty. Roney Jone P. Gandeza (a) If the two things can be separated without injury, their respective owners may demand their separation. (Art. 469, par. 1 Civil Code) (b) If the two things cannot be separated without injury, and both the owners had acted in good faith, the owner of the principal thing acquires the accessory indemnifying the owner of such accessory for its value (Art. 456, Civil Code). Nevertheless, if the accessory thing is much more precious than the principal thing, the owner may demand its separation, even though the principal thing may suffer some injury. (c) If the owner of the accessory thing has made the incorporation in bad faith, he loses the thing incorporated and shall have the obligation to indemnify the owner of the principal thing for the damages which the latter may have suffered. (d) If the one who has acted in bad faith is the owner of the principal thing, the owner of the accessory thing may choose between the former paying him its value or that his accessory thing be separated, even though it will cause damage or injury to the principal. Moreover, the owner of the principal thing shall be liable for damages. (e) If both owners had acted in bad faith, their respective rights shall be determined as though both had acted in good faith. 149. What is meant by rebus sic stantibus?

Rebus sic stantibus is a rule which provides that an agreement is valid only if the same conditions prevailing at the time of contracting continue to exist at the time of performance. 150. What is the nature of reformation of instruments as a remedy in law? Reformation of instruments (not reformation of contracts) is a remedy to conform to the real intention of the parties due to mistake, fraud, inequitable conduct, or accident. The action prescribed in 10 years from date of execution of instrument. 151. What nuisance?

is

an

An attractive nuisance is a dangerous instrumentality or appliance which is likely to attract children at play. 152. What is the doctrine attractive nuisance?

of

One who maintains on his estate or premises an attractive nuisance without exercising due care to prevent children from playing therewith or resorting thereto, is liable to a child of tender years who is injured thereby, even if the child is technically a trespasser in the premises. The reason for the doctrine is that the attractiveness is an invitation to children. Safeguards to prevent danger must be set up. 153. X was coerced into marrying Y. X sued for annulment. During the pendency of the case, X married Z. When Z learned of the first marriage, Z

I press toward BAR OPERATIONS 2013 the mark for the

prize of the high calling of God in Christ Jesus. - Philippians 3:14

BARRISTERS’ CLUB

attractive

Page 50 of 67

in CIVIL LAW Prepared by: Atty. Roney Jone P. Gandeza sued X for bigamy. X now alleges that the pendency of the annulment case is a prejudicial question. Is X correct? X is wrong because the decision in the annulment case is not important. The first marriage will either be annulled or not. If not annulled, bigamy can prosper. If annulled, still bigamy can prosper, for when he married the second time, he was still married to his wife, a voidable marriage being considered valid until annulled. 154. X, a married man, was forced by Y to contract marriage with her. X then sued for annulment of the second marriage; Y retaliated with a charge of bigamy. In the bigamy case, X moved to suspend the criminal proceedings until after the termination of the annulment case on the ground that the annulment case is a prejudicial question. Should the motion be granted? Yes, because the annulment case poses a prejudicial question. If X was really forced into marrying Y, then his consent was defective; hence, the second marriage is to be annulled on that ground. He cannot therefore be guilty of bigamy. 155. In his will, testator T a) disinherits his daughter, A, because “she married a food for nothing gigolo despite my repeated warnings that she shouldn’t marry him”; b) omits his wife, W; c) leaves a legacy of P10, 000 to his mistress, M, and P5, 000 to his driver, E; and d)

institutes his son, B, as his sole heir. Distribute T;s estate of P100, 000. The disinheritance of A was ineffective because the ground relied upon by T does not constitute a valid ground for disinheritance under Article 919 of the Civil Code. Hence, the testamentary provisions in the will shall be annulled but only to the extent that A’s legitime was impaired. The total omission of W does not constitute preterition because she is not a compulsory heir in the direct line. Only compulsory heirs in the direct line may be the subject of preterition. Not having been preterited, she is entitled to her legitime. The legacy in favor of M is void under Article 1028 of the Civil Code for being in consideration of her adulterous relations with T. She is, therefore, disqualified to receive the legacy of P10, 000. The legacy of P5, 000 in favor of E is not inofficious because it does not exceed the free portion. Hence, E shall be entitled to receive it. The institution of B, which applies only to the free portion, shall be respected. In sum, the estate of T will be distributed as follows: A B W E M

25, 000 45, 000 25, 000 5, 000 0

I press toward BAR OPERATIONS 2013 the mark for the

prize of the high calling of God in Christ Jesus. - Philippians 3:14

BARRISTERS’ CLUB

Page 51 of 67

in CIVIL LAW Prepared by: Atty. Roney Jone P. Gandeza 156. Within what period may an action to revoke a donation inter vivos be filed by the donor? An action to revoke a donation inter vivos must be filed by the dono within four years if the ground is the subsequent birth, adoption or reappearance of a child of the donor or non-fulfillment of a condition; and it must be filed within one year id the ground is act of ingratitude in the part of the donee. As to the first two grounds, the right to file the action is transmitted to the heirs; as to the last ground only the donor can file the action and the right is not transmitted to the heirs. 157. What title?

is

a

muniment

of

A muniment of title is an instrument or written evidence which an application for land registration holds or possesses to enable him to substantiate and prove title to his estate. 158. When is pendens not applicable?

a notice registrable

lis or

A notice lis pendens is not registrable or applicable to the following: (a) preliminary attachment; (b) proceedings for the probate of wills; (c) levies on execution; (d) proceedings for administration of estate of deceased persons; and (e) proceedings in which the only object is the recovery of sum of money. 159. Does a previous final judgment denying a petition for declaration of nullity of marriage on the ground of psychological incapacity bar a subsequent petition for annulment on the

ground of license?

lack

of

Yes. In both petitions, the cause of action is the same – the declaration of nullity of marriage. What differs is only the ground upon which the case of action is predicated. A party cannot avoid the application of res judicata by simply varying the form of his action or adopting a different method of presenting his case. (Mallion v. Alcantara, GR No. 141528, October 31, 2006) 160. Articles 238 to 253 of the Family Code govern “Summary Judicial Proceedings in the Family Law.” Are these proceedings governed by the Revised Rule on Summary Procedure? No. These summary judicial proceedings fall within the jurisdiction of the RTC, while the cases covered by the Revised Ruled on Summary Procedure are cases falling within the jurisdiction of the Municipal Trial Court. These summary judicial proceedings are in fact governed by the special rules in Articles 239 to 253 of the Family Code. Thus, in the proceedings to secure the consent of the estrange spouse to a transaction, a preliminary conference is to be conducted by the judge without the parties being assisted by counsel; while under the Revised Rules on Summary Procedure, the parties who are required to be present in the preliminary conference, are assisted by counsel. 161. A person who is a beneficiary in a will is competent to act as an instrumental witness in the execution of a will. Is this statement accurate?

I press toward BAR OPERATIONS 2013 the mark for the

prize of the high calling of God in Christ Jesus. - Philippians 3:14

BARRISTERS’ CLUB

marriage

Page 52 of 67

in CIVIL LAW Prepared by: Atty. Roney Jone P. Gandeza Yes, but his institution as an heir, or the legacy or devise given to him, shall be rendered void, unless there are three other competent witnesses (Art. 823, CC). In other words, he is disqualified from inheriting from the testator (Art. 1027, CC). 162. If a marriage is dissolved because of the death of the husband, what surname may the widow use? Although the death of the husband dissolves the marriage ties, still the window may desire to cherish her deceased husband’s memory by the continued use of his surname. However, if she does not want to, she is allowed to use her maiden surname. Notice the use of the word “may” in Article 373 of the Civil Code. 163. If a marriage is annulled, is the wife required to resume her maiden name and surname? Wife is the guilty spouse: She SHALL resume her maiden name and surname. Wife is the innocent spouse: She MAY resume her maiden name and surname, but she may choose to continue employing her former husband’s surname, unless (1) the court decrees otherwise; or (2) she or the former husband is married again to another person. (Art. 371, CC) 164. What if legal separation occurs, is the wife entitled to continue using the husband’s surname even of she is the guilty spouse?

Article 372 of the Civil Code does not distinguish whether the wife is the guilty spouse or not, unlike in the case of annulment of marriage under Article 371, because in legal separation the marriage ties still subsist. 165. D owes C P30, 000. D goes to C and tells the latter that he is proposing X as the new debtor who will pay the debt. C accepts the substitution. If X later becomes insolvent, may C demand payment of the obligation from D? What obtains in this problem is delegacion. In delegacion, the debtor offers, and the creditor accepts, a third person who consents to the substitution. Pursuant to Article 1295 of the Civil Code, the insolvency of X who has been proposed by D and accepted by C, shall not revive C’s action against D, the original obligor, except when said insolvency already existed and of public knowledge, or known to the debtor when he delegated his debt. 166. Suppose the insolvency of X is of public knowledge but not known to D and the creditor who also does not know, gives his consent, may C hold D liable? Yes, because the initiative came from the debtor who should know the economic or financial status of his substitute or delegado. 167. If an agreement falling within the Statute of Frauds is made in writing and duly subscribed but the same is subsequently lost or destroyed, is the agreement judicially enforceable?

I press toward BAR OPERATIONS 2013 the mark for the

prize of the high calling of God in Christ Jesus. - Philippians 3:14

BARRISTERS’ CLUB

Page 53 of 67

in CIVIL LAW Prepared by: Atty. Roney Jone P. Gandeza Yes, because the first (introductory) part of the Statute of Frauds (Art. 1403, par. 2 Civil Code) states that if the agreement is not in writing and duly subscribed, evidence of such agreement cannot be received without the writing, or a secondary evidence of its contents. Thus, the loss or destruction of a written agreement (complying with the Statute of Frauds) will not militate against the validity or enforceability of the agreement. The agreement us valid and enforceable and its existence and the contents thereof can be proved by secondary evidence like the testimony of the interested part and his witness. (see Sec. 4, Rule 130, Rules of Court) 168. A orally sold to B his land. Later B wanted to have the land registered, but registration requires a public instrument. May B compel A to execute the needed public instrument? It depends. If the contract is still executor, B cannot compel A to execute the notarized sale because the contract is unenforceable. However, if the price has been paid, or the land has been delivered, this time, B can compel A because the contract is both valid and enforceable. QUESTIONS 169-170 based on the following situation:

are fact

Among the items inherited by B from his mother were some old paintings that had been stored in his mother’s attic for a number of years. B knew nothing about art and had no place to put them in his house. He placed an

ad in the paper offering to sell the paintings “at a price to be mutually agreed upon”. C, a buyer for an art gallery, responded to the ad and examined the paintings. From the signature and the style, C recognized that the artist was BenCab, a renowned Filipino portrait artist. A and B agreed upon a price and signed a contract. 169. Which of the following facts, if true, would give B the best basis for avoiding the contract with C? a. B told C that his mother had been dabbled in painting when she was younger and had undoubtedly painted them herself. b. B did not know that C was a buyer for an art gallery and was very familiar with the works of renowned Filipino artists. c. B told C that he wanted to get rid of the paintings as soon as possible because he was angry at his mother for giving away most of her possessions to her friend just before he died. d. C falsely told B that the paintings were to be used to furnish C’s newlyconstructed vacation house in Tagaytay City. The statement in (A) would enable B to annul the contract on the ground of mistake if C was aware that B was mistaken about the identity of

I press toward BAR OPERATIONS 2013 the mark for the

prize of the high calling of God in Christ Jesus. - Philippians 3:14

BARRISTERS’ CLUB

Page 54 of 67

in CIVIL LAW Prepared by: Atty. Roney Jone P. Gandeza the artist. Under the facts in this choice, C knows that B is mistaken about the identity of the artist, which mistake refers to the substance of the thing which is the object of the contract. The statement in (B) is wrong because the fact that one of the parties to the contract has superior knowledge about the subject matter of the contract does not by itself justify annulment, even of the other party is unaware of that fact. C’s knowledge or lack of it was not the principal cause on which the contract was made and was not relied on by B in making the sale. The statement in (C) is wrong because the fact that B was angry when he agreed to the contract is not a ground for annulment of a contract under the law. Regardless of B’s state of mind, there was a meeting of the minds between the parties. The statement in (D) is incorrect because C’s misrepresentations to B as to how she will use the paintings does not appear to go to the substance of the thing which is the subject matter of the contract or to have been relied on by B. Hence, the misrepresentation is not significant enough to serve as a ground for annulling the contract. 170. Which of the following facts, if true, would give C the best basis for annulling the contract? a. Several of the paintings cracked when they were being transported by V because they were brittle with age.

b. The day after the purchase, a respected art historian announced in a press release that several of X’s paintings were actually done by his students, causing the value of all X’s paintings to decline. c. Because of some experimental pigments that the artist had used, the colors began to fade rapidly as soon as the paintings were exposed to light; within a few days, virtually all of the colors had faded away. d. The gallery for which C had procured the paintings was destroyed by a fire shortly after the contract was executed. (C) offers the best ground for annulling the contract based on mutual mistake. When both parties entering into a contract are mistaken about facts relating to the agreement, the contract may be avoidable by the adversely affected party if the latter did not assume the risk or contingency of mistake. Here, both parties believed that the paintings would be suitable for viewing and had no reason to suspect that their color would fade away when exposed to light. Despite C’s superior knowledge of the subject matter of the contract, it is doubtful that she would be deemed to have assumed the risk or contingency of what occurred to the paintings. The circumstances in (A) are not that strong as basis for annulment.

I press toward BAR OPERATIONS 2013 the mark for the

prize of the high calling of God in Christ Jesus. - Philippians 3:14

BARRISTERS’ CLUB

Page 55 of 67

in CIVIL LAW Prepared by: Atty. Roney Jone P. Gandeza Even assuming that both parties mistakenly believed that the paintings were not too fragile to be transported, that risk is more likely to be assumed by C.

The preterition of C annuls the institution of A, B and F as T’s heirs. Intestacy results. Therefore, A, B and C will each get P30, 000. The friend, F, get nothing.

The circumstances in (B) suggest only that C has made a mistake as to the value of the paintings, but since B knew nothing about the identify of the artist, the mistake is unilateral and C cannot annual on this ground.

173. Testator T has three legitimate children: A, B and C. In his will, T disinherits A and institutes B and C as his heirs. The disinheritance of A is invalid because it is for a cause not provided by the law. If the hereditary estate is P90, 000, how shall the distribution be made?

Choice (D) is incorrect because the circumstances do not satisfy the requirements for annulment. Under Article 1351 of the Civil Code, the particular motives of the parties in entering into the contract are different from the cause thereof. 171. W, wife of H, institutes a petition seeking a judicial declaration of presumptive death of her husband who has been missing since 2005. Will the petition prosper? It depends. If the action is to enable W to contract a subsequent marriage, the petition is open the estate of H to succession or for other purpose, the petition will not prosper because the presumption is already established by law. 172. Testator T has three legitimate children, A, B and C. T made a will instituting his children, A and B, and a friend, F, as his sole heirs. C was completely omitted in the inheritance. If T’s estate at the time of his death is P90, 000, how should the distribution be made?

The institution of B and C remains valid, but their shares are to be reduced to give A his legitime. Had there been preterion here, each would receive P30, 000 each. Therefore, A is entitled to his legitime of P15, 000; B to his legitime of P15, 000 plus his share as an instituted heir in the amount of P22, 500; and C is entitled to the same share as B. 174. X borrowed from his friend, Y, the latter’s car for an out of town trip. On his way to his destination, X met with an accident which greatly damages the car. X was not at fault because he was driving carefully. If the car is to be repaired, the expenses for repair shall be borne by: a. X alone, whether he is at fault or not. b. Y alone, because X was not at fault. c. Y alone, he being the owner of the car.

I press toward BAR OPERATIONS 2013 the mark for the

prize of the high calling of God in Christ Jesus. - Philippians 3:14

BARRISTERS’ CLUB

Page 56 of 67

in CIVIL LAW Prepared by: Atty. Roney Jone P. Gandeza d. X any Y in equal shares, the expenses being deemed extraordinary. The choice in (D) is correct because the expenses are deemed ordinary, and not due to the wear and tear of the thing. This is pursuant to the second paragraph of Article 1949 of the Civil Code which provides that if the extra-ordinary expenses arise from the actual use of the bailee of the thing loaned, even though he acted without fault, the expense is borne equally by the bailor and the bailee share and share alike. 175. A borrowed B’s truck. During a fire which broke out in A’s garage, he had time to save only one vehicle, and he saved his car instead of the truck. Is A liable for the loss of B’s truck? Yes. The baille in commodatum is liable for the loss of the thing loaned, even if it should be through a furtuitious event, if being able to save either the thing borrowed or his own thing, he chose to save the latter. (Art. 1492, CC) 176. D borrowed money from C. To guarantee payment, D left the Torrens title of his land to C for the latter to hold until payment of the loan. Is there a: a) contract of pledge?

CC) If at all, there was a pledge of the paper or document constituting the Torrens Title, as a movable by itself, but not of the land which the title represents. There is no mortgage because no deed or contract was executed in the manner required by law for a mortgage. There is no contract of antichresis because no right to the fruits of the property was given to the creditor. (Art. 2131, CC) 177. A leased a building to B for a term of ten years. B established a shoe factory in the building and installed therein certain machineries. Are the machineries movables or immovables? The machineries are movables. Machineries are deemed immovables only if: a) they are placed by the owner of the tenement. b) an industry or works is carried on in the tenement. c) machinery is intended for such industry or works. d) machinery tends directly to meet the needs of such industry or works. In the problem presented, the machineries are movables since they were installed by B, and not by A.

b) contract of mortgage? c) contract of antichresis? d) none of the above? None of the above. There is no pledge because only movable property may be pledged. (Art. 2094,

178. Suppose the lessee promised to leave the machineries to the lessor at the end of the lease, would that make any difference in your answer?

I press toward BAR OPERATIONS 2013 the mark for the

prize of the high calling of God in Christ Jesus. - Philippians 3:14

BARRISTERS’ CLUB

Page 57 of 67

in CIVIL LAW Prepared by: Atty. Roney Jone P. Gandeza Yes, because the machineries would then be considered as immovables, because the tenant would then be considered as the agent of the lessor. 179. A is the owner of a piece of land upon which fruits were grown, raised, harvested, and gathered by B in bad faith. Who owns the fruits? A, the landowner, owns the fruits with no obligation to indemnify B, except the latter’s expense in the production, gathering and preservation of the fruits. This is pursuant to the rule that no one may unjustly enrich himself at the expense of another. (Art. 442, CC) ARTICLE 449: crops have not yet (here the landowner without indemnity by accession continua)

applies if the been gathered gets the fruits the principle of

ARTICLE 443: applies when the crops have already been gathered. Article 443 does not apply when the planter is in good faith, because under Article 544, he is already entitled to the fruits received and there is no necessity of reimbursing him. 180. X, who was abroad, phoned his brother, Y, authorizing him to sell X’s property in Pasay. X sent the title to Y by courier service. Acting for his brother, Y executed a notarized deed of absolute sale of the land to Z after receiving payment. The sale is: a) valid, if the sale was notarized and buyer paid in full.

b) void, because X should have authorized Y in writing to sell the land. c) valid, because Y was his brother X’s agent and entrusted with the title needed to effect the sale. d) enforceable, because the buyer could file an action to compel X to execute a deed of sale. (B) is correct because of the requirement that the sale of realty through an agent must be in writing to be valid. In the problem presented, the authority given by X to Y was not in writing; hence, the sale is void. 181. Paterno gave a power of attorney to Ambrosio authorizing him to sell Paterno’s car for Php200, 000 payable in cash. Under these circumstances, what are the consequences of Ambrosio’s appointment by Paterno as the latter’s agent to sell his car? The authority of Ambrosio to sell the car is express. It includes the implied authority to receive payment and to give a receipt as they are acts necessary to accomplish the purpose of the agency. They are both actual authority. If Paterno privately instructed Ambrosio not to consummate the sale, the sale by Ambrosio is still binding upon Paterno as Ambrosio had apparent or ostensible authority to sell. As far as third persons are concerned, the effect is as if Ambrosio had actual authority.

I press toward BAR OPERATIONS 2013 the mark for the

prize of the high calling of God in Christ Jesus. - Philippians 3:14

BARRISTERS’ CLUB

Page 58 of 67

in CIVIL LAW Prepared by: Atty. Roney Jone P. Gandeza The same is true if Paterno had not authorized Ambrosio to sell the car but having knowledge that Ambrosio was acting for him kept silent and after consummation of the sale, received the proceeds thereof from Ambrosio. Here, Ambrosio’s authority rests on estoppels on the part of Paterno to deny such authority. The authority given to Ambrosio to sell the car is special because it involves a particular transaction. Ambrosio has no authority to use the car for his own purposes but he can use it in an emergency as, for example, to take a member of his family to a hospital. In this case, his authority is demanded by necessity. 182. Give examples of agency by estoppels and implied agency. AGENCY BY ESTOPPEL: P tells X that A is authorized to sell certain merchandise. P privately instructs A not to consummate the sale but merely to find out the highest price X is willing to pay for the merchandise. If A makes a sale to X, the sale is binding on P who is in estoppels to deny A’s authority. In this case, there is no agency created but there is a power created in A to create contractual relations between P and a third person, without having authority to do so. The legal result is the same as if A had authority to sell. IMPLIED AGENCY: P authorized A to sell his car. A sold the car to X who paid the purchase price. However, A did not give the money to P. X is not liable to P. A has implied authority to receive payment.

183. Under Article 1772 of the Civil Code, “every contract of partnership having a capital of 3, 000 or more, in money or property, shall appear in a public instrument, which must be recorded in the office of the Securities and Exchange Commission.” Suppose this requirement has not been complied with, is the partnership still a juridical person, assuming that all other requisites are present? Yes. Article 1768 of the Civil Code provides that “the partnership has a juridical personality separate and distinct from that of each of the partners, even in case of failure to comply with the requirements of Article 1772.” It is settled that the registration under Article 1772 of the Civil Code is not intended as a prerequisite for the acquisition of juridical personality by the partnership, but merely as a condition for the issuance of the license to engage in business or trade. 184. X, Y and Z entered into partnership under the firm name “XYZ Ltd”. Upon mutual agreement, X withdrew from the partnership and the partnership was dissolved. However, the remaining partners, Y and Z, did not terminate the business. Instead of winding up the business of the partnership and liquidating its assets, Y and Z continued the business in the name of “XYZ Ltd.” Apparently without the objection of X. The withdrawal of X from the partnership was not published in the papers.

I press toward BAR OPERATIONS 2013 the mark for the

prize of the high calling of God in Christ Jesus. - Philippians 3:14

BARRISTERS’ CLUB

Page 59 of 67

in CIVIL LAW Prepared by: Atty. Roney Jone P. Gandeza Is X liable for any obligation Y and Z might incur while doing business in the name of “XYZ Ltd.” after his withdrawal from the partnership? Yes, X can be held liable under the doctrine of estoppels. But as regards the parties among themselves, only Y and Z are liable. X can be held liable since there was no proper notification or publication to the public in general. In the event that X is made to pay the liability to a third person, he has the right to seek reimbursement from Y and Z. (Arts. 1837 to 1849, CC) 185. A, B and C are general partners in s trading frim. Having contributed equal amounts to the capital, they agreed on equal distribution of whatever net profile is realized per fiscal period. After four years of operation, A conveyed his whole interest in the partnership to D without the knowledge and consent of B and C. Is the partnership dissolved? No. Under Article 1813 of the Civil Code, the conveyance by a partner of his or her whole interest in the partnership does not itself dissolve the partnership on the absence of an agreement. 186. What are the rights of D shoule he desire to participate in the management of the partnership and in the distribution of the net profit of P45, 000 which was earned after his purchase of A’s interest?

participate in the management or administration of the partnership business affairs. He may, however, receive the net profits to which A would have otherwise been entitled. In This case, D is entitled to 1/3 of the profits. 187. Due to a misunderstanding, Atty. Dumag withdraws his membership from a law firm which does not have a fized term. He later files with the SEC a petition for the dissolution of the firm. After due proceedings, the SEC hearing officer denied the petition on the ground that the withdrawal of Atty. Dumag has not dissolved the firm Is the SEC hearing officer correct? No. A partnership which does not have a fixed term is a partnership at will. The law firm Atty. Dumag is one such partnership. The birth and life of a partnership at will is predicated on the mutual desire and consent of the partners. The right to choose with whom a person wishes to associate himself is the very foundation and essence of that partnership. Its continued existence is, in turn, dependent on the constancy of that mutual resolve, along with each partner’s capability to give it. Verily, anyone of the partners may, at his sole pleasure, dictate dissolution of the partnership at will. He must, however, act in good faith. If he acted in bad faith, it would not prevent the dissolution of the partnership but that it can result in a liability for damages. (Ortega, et.al., v. Court of Appeals, 245 SCRA 529 [1995])

Also pursuant to Article 1813 of the Civil Code, D cannot interfere nor

I press toward BAR OPERATIONS 2013 the mark for the

prize of the high calling of God in Christ Jesus. - Philippians 3:14

BARRISTERS’ CLUB

Page 60 of 67

in CIVIL LAW Prepared by: Atty. Roney Jone P. Gandeza 188. A, B and C are partners in a construction firm. Due to a quarrel, C withdrew from the partnership in 1986 as a result of which they agreed to dissolve their partnership and executes an agreement of partition and distribution of the partnership properties. In 1994, C’s heirs brought an action against A and B for accounting of the partnership assets and partition. A and B defended on the ground of prescription. They contend that the action had already prescribed four years after it accrued in 1986 when the partnership was dissolved by the withdrawal of C. Has the action prescribed? Contrary to A and B’s allegation, prescription has not even begun to run in the absence of a final accounting. Article 1842 of the Civil Code states that the right to demand accounting accrues at the date of dissolution in the absence of any agreement to the contrary. When a final accounting has been made, it is only then that prescription begins to run. In the case at bar, no final accounting has been made, and that is precisely what the heirs of C are seeking since A and B failed or refused to render an accounting of the partnership’s business and assets. Hence, the action is not barred by prescription. (Emnace v. Ca, 370 SCRA 431) 189. C loans D the amount of P100, 000. As security for the loan, D delivered to C two rings by way of pledge. When D failed to pay, C foreclosed, and had the rings sold at auction. The

proceeds of the sale, after deducting expenses, amounted only to P70, 000. Is C entitled to demand the deficiency from D? No. The Civil Code on pledge provides that the foreclosure of the pledge extinguishes the principal, whether or not the proceeds of the sale are more, or less than the amount of the principal obligation. (Art. 2115, CC) Under Article 2115 of the Civil Code, if the price of the sale is less than the principal obligation, the creditor shall not be entitled to recover the deficiency, notwithstanding any stipulation to the contrary. 190. What if the rings were sold for the net amount of P150, 000, is D entitled to the excess? Under Article 2115 of the Civil Code, if the price of the sale is more than the amount of the principal obligation, the debtor shall not be entitled to the excess, unless it is otherwise agreed upon. (Art. 2115, CC) 191. Suppose the rings had been chattel mortgaged to C, is D liable to pay the deficiency if the proceeds of the sale were less than the indebtedness? Or, is she entitled to the excess, if the proceeds were more? If instead of a pledge the rings were given by way of a chattel mortgage, C can recover the deficiency from D as there is no prohibition in the Chattel Mortgage Law similar to pledge. The excess, if

I press toward BAR OPERATIONS 2013 the mark for the

prize of the high calling of God in Christ Jesus. - Philippians 3:14

BARRISTERS’ CLUB

Page 61 of 67

in CIVIL LAW Prepared by: Atty. Roney Jone P. Gandeza any, should be returned to D, the mortgagor. 192. Lessor and Lessee verbally agree that if Lessor should decide to sell the building, he will give Lessee the pre-emptive right to buy the leased property. The following year, Lessor offered to sell to Lessee the building for P2 million, but Lessee counteroffered for P1 million. Lessee later learned that a buyer had already purchased the property from Lessor for P800, 000. He also discovered that the sale had already been consummated when Lessor offered to sell it to him. Lessee thus offered to reimburse the buyer the purchase price of P800, 000, plus an additional P200, 000 to complete his earlier offer of P1 million. When the offer was refused, Lessee brought an action for rescission of the sale. After due proceedings, however, the court dismissed the complaint on the ground that “the right of redemption on which the complaint is based is merely an oral one as such, is unenforceable under the law.” Is the right governed by the Frauds?

of refusal Statute of

No. A right of first refusal is not among those listed as unenforceable under the Statute of Frauds. The application on Article 1403, par. 2(e) of the Civil Code presupposes the existence of a perfected contract of sale of real

property. It is a contractual grant, not of the sale of real property involved but of the right of first refusal over the property sought to be sold. The right of first refusal need not be written to be enforceable and may br proved by oral evidence. (Rosencor Devt. V. Inquing, 354 SCRA 119) 193. T died in 1990 with a will. In his will, he devised a house and lot to his friend, A, as first heir and to C, A’s son, as second heir. B died in 1995 survived by his two children E and F. A himself died in 2000 survived by his children C and D. In the settlement of A’s estate, E and F filed a motion to exclude the house and lot originating from T on the ground that they are the exclusive owners of the property. C and D opposed the motion on the ground that B, the second heir, predeceased A, and that therefore, the fideicommissary substitution did not produce any effect as far as B, the second heir, is concerned. Should the opposition be sustained? No. B, the second heir, acquires a right to the succession from the time of the testator’s death, even though he, B, should die before the fiduciary, A. B inherited from T as second heir when the latter died in 1990. When B died in 1995, he was able to transmit his right to his own heirs, E and F.

I press toward BAR OPERATIONS 2013 the mark for the

prize of the high calling of God in Christ Jesus. - Philippians 3:14

BARRISTERS’ CLUB

Page 62 of 67

in CIVIL LAW Prepared by: Atty. Roney Jone P. Gandeza When A (first heir) died in 2000, the right of E and F over the property became absolute. 194. H died leaving an estate of P100, 000. His widow, W, gave birth to a child four months after H’s death, but the child died five hours after birth. Two days after the child’s death, W also died because she had suffered from difficult child birth. The estate of H is now being claimed by his parents, A and B, and by C and D, the parents of W. Who is entitled to H’s estate of P100, 000? If the child had an intra-uterine life of not less than seven months, it inherited from the father. Consequently, the estate of P100, 000 shall be divided equally between the child and his mother as legal heirs. Upon the death of the child, its share of P50, 000 goes by operation of law to the mother, W, which is subject to reserve troncal. Under Article 891 of the Civil Code, the reserve is in favor of relatives belonging to the paternal line and who are three degrees from the child. The parents of H (A and B) are entitled to the reserved portion which is P50, 000 as they are two degrees related from the child. The P50, 000 inherited by W from H will go to her parents, C and D, as her legal heirs. However, if the child had an intra-uterine life of less than seven months, half of the estate of H, or P50, 000, will be inherited by W, the widow while the other half, P50, 000, will be inherited by the parents of H. Upon the death of W, her estate of P50, 000 will be inherited by her own parents, C and D.

195. Before his death in 1990, A donated to his grandson, F, a child of his predeceased son D, a house and lot worth P600, 000. In 1995, F died with a will instituting his mother, E, as his sole heir. His estate consisted entirely of the house and lot which he had received from A. In 1998, E also died but without a will. The house and lot is now claimed by: (a) B, widow of A and grandmother of F; (b) C, son of A and B uncle of F; and (c) G, sister and only living relative of E. To who, shall the property be adjudicated? Half to C; other half to G. F, the propositus, died with a will instituting his mother, E, as sole heir. Consequently, only one0half of the property passed to her by operation of law since that is her legitime. Only this portion of the property has become reservable. B is not entitled to the reservable portion of the property. Although a relative of the propositus in the second degree, B is merely related by affinity to the ascendant (A) from whom the property came. She does not, therefore, belong to the “line from which the property came.” A reservatario must not only be related by consanguinity to the propositus within the third degree, but he must also be related by

I press toward BAR OPERATIONS 2013 the mark for the

prize of the high calling of God in Christ Jesus. - Philippians 3:14

BARRISTERS’ CLUB

Page 63 of 67

in CIVIL LAW Prepared by: Atty. Roney Jone P. Gandeza consanguinity to the ascendant from whom the property came. C is entitled to the reservable portion of the property since he is not only a third degree relative by consanguinity of the propositus, but he also belongs to the line from which the property came.

FIRST, the property had been acquired by operation of law by an ascendant (F)from his descendant (Y) upon the death of the latter. SECOND, the property had been previously acquired by gratuitous title by the descendant (Y) from a brother (X).

G, on the other hand, is entitled to the part of the property which is not reservable in accordance with the ordinary rules of intestate succession. 196. D, only daughter of B, married E, only son of A, in 1981. A son, X, was born to the couple in 1982. E died in a vehicular accident in 1984. In 1986, D married F, only son of C. A son, Y was born to the couple in 1988. D also died in a vehicular accident in 1992. In 1995, X, who was very sickly, donated to his half brother, Y, a parcel of land. X died the following year. In 1998 Y also died. He died intestate and without any surviving issue. The land which he had acquired from X was inherited by his father, F, who was his only legal heir. F died intestate in 2002, survived only by his father, C. The land which had originated from X is now being claimed by A and B on the ground that it is reservable. C claims that the property belongs to him and him alone as his inheritance from his son, F.

THIRD, descendant (Y) died without any legitime issue who can inherit from him.

Who among the grandfathers is entitled to the property?

EXCEPT FOR CHATTEL MORTGAGE, a pledge, real estate mortgage, or antichresis may exceptionally secure “after-incurred” obligations so long as these future debts are accurately described. This is so because chattel mortgages can

B alone is entitled to the property. The property is reservable.

In order to determine who can qualify as a reservatario, two test should be applied: First is the claimant a relative of the descendantpropositus within the third degree? Second, does he belong to the line from which the reservable property came? Applying the tests, it is clear that: Neither can C qualify because he does not belong to the line from which the property came. He is not related by consanguinity to X. Only B can qualify. He is not a relative of Y within the third degree; he also belongs to the line from which the reservable property came. 197. May a contract of pledge, real estate mortgage, antichresis and chattle mortgage secure “after-incurred” obligation?

I press toward BAR OPERATIONS 2013 the mark for the

prize of the high calling of God in Christ Jesus. - Philippians 3:14

BARRISTERS’ CLUB

Page 64 of 67

in CIVIL LAW Prepared by: Atty. Roney Jone P. Gandeza only cover obligations existing at the time the mortgage is constituted. One of the requirements of chattel mortgage is an affidavit of good faith and the law has provided that the parties to the contract must execute an oath that the mortgage is made for the purpose of securing the obligation specified in the conditions thereof and for no other purposes. The debt referred to in the law is a current obligation, not an obligation that is merely contemplated. 198. Which irregularities on formal requisites of marriage would not affect the validity of a marriage but may render the party or parties civilly, criminally, or administratively liable? a) Marriage license is issued by a local civil registrar of a city or municipality where neither party habitually resides. b) Marriage license is presigned by a local civil registrar but issued by a fixer. c) Marriage is performed at a place other than those designated by law without the written request under oath of the parties. d) No marriage counseling certificate is attached to the application and the marriage license is issued even before the lapse of the three0month period, the marriage is valid. e) Marriage ceremony is performed with only one witness present or with no witnesses at all (“secret marriage”).

f) Marriage license is issued without waiting for the lapse of the three-month period. g) Marriage license is issued without waiting for the lapse of the ten-day period, or if the notice of application is not published. h) Marriage remains valid neither contracting party belongs to the solemnizing officer’s church or religious sect. i) The solemnizing officer is not duly registered with the civil registrar general at the time of the marriage. j) The solemnizing officer solemnized the marriage in violation of the written authority granted to him by his church or religious sect, the marriage is valid. k) A judge marriage outside jurisdiction.

solemnizes a his court’s

l) Alien spouse obtains a marriage license without a certificate of legal capacity. 199. If a party has been previously married, and the prior marriage has been dissolved, is parental consent still required if such party is still below 21 years of age? Not anymore. This is so because of Article 14 of the Family Code which provides, “In case either or both of the contract parties, not having been previously emancipated by a previous marriage, x x x.” This means parental consent is no longer required for these parties.

I press toward BAR OPERATIONS 2013 the mark for the

prize of the high calling of God in Christ Jesus. - Philippians 3:14

BARRISTERS’ CLUB

Page 65 of 67

in CIVIL LAW Prepared by: Atty. Roney Jone P. Gandeza 200. What is possession?

a

mortgage

in

A mortgage in possession, otherwise known as antichresis, is one where the mortgagee acquires actual or constructive possession of the property mortgaged for purposes only of enforcing his security over the property and collecting the income to pay for the mortgagor’s debt. (Nadal v. CA, 320 SCRA 699)

unless such usufruct has been granted only in consideration of the existence of such person. If the document constituting V as a usufructuary does not state that it will end the moment Mrs. X is dead, then it will continue.

201. A and B are childhood friends. Because of their friendship, A gave to B in usufruct a parcel of land to last up to the time their high school teacher, Mrs. X, reaches the age of 70. Mrs X died at the age of 65. Is the usufruct extinguished? No. Under Article 606 of the Civil Code, a usufruct granted for a time that may elapse before a third person attains a certain age shall subsist from the number of years specified even if the third person should die before the period expires,

I press toward BAR OPERATIONS 2013 the mark for the

prize of the high calling of God in Christ Jesus. - Philippians 3:14

BARRISTERS’ CLUB

Page 66 of 67

THE BARRISTERS’ CLUB OFFICERS Chancellor: Vice Chancellor:

ABBYGAILE T. GONZALES ROMEL L. BASILAN

Secretary:

JESSA ALYSSA G. REYES

Treasurer:

MILDRED P. AMBROS

PRO:

ROBYN B. DELA PENA

PRO:

AARON JAMES E. CO

Business Manager:

RUDDY ALLEN N. YEE

Business Manager:

LESLIE D. RAGUINDIN

SSG Representative: Ex-Officio:

ANNE LUCILLE B. RUIZ

Adviser: Dean, College of Law:

RONA B. ESTRADA ATTY. ISAGANI G. CALDERON ATTY. REYNALDO U. AGRANZAMENDEZ

View more...

Comments

Copyright ©2017 KUPDF Inc.
SUPPORT KUPDF